Quick viewing(Text Mode)

Test - 15(Mock Test - 12)

Test - 15(Mock Test - 12)

TEST - 15(MOCK TEST - 12)

1 Consider the following statements. 1. UN peacekeeping operations are deployed on the basis of mandates from the United Nations Security Council. 2. The UN Charter gives the Security Council primary responsibility for the maintenance of international peace and security. 3. All UN Member States share the costs of peacekeeping, but the Security Council is required to pay a larger share. 4. The General Assembly approves and oversees the peacekeeping budget.

Select the correct answer using the codes below. A. 1 and 2 only B. 2 and 3 only C. 1, 3 and 4 only D. 1, 2, 3 and 4 only

Your Answer : Correct Answer : D

Answer Justification :

Justification: Statement 1: Chapter VI of the UN Charter deals with the “Pacific Settlement of Disputes”. UN peacekeeping operations have traditionally been associated with Chapter.

However, the Security Council need not refer to a specific Chapter of the Charter when passing a resolution authorizing the deployment of a UN peacekeeping operation and has never invoked Chapter VI.

Statement 2: In fulfilling this responsibility, the Council can establish a UN peacekeeping operation.

Statement 3: As all UN Member States share the costs of peacekeeping, the Assembly apportions these expenses based on a special scale of assessments, taking into account the relative economic wealth of Member States, with the permanent members of the Security Council required to pay a larger share because of their special responsibility for the maintenance of international peace and security.

Statement 4: While not normally directly involved in political decisions on establishing or terminating UN Peacekeeping operations, the General Assembly does play a key role in peacekeeping financing.

The General Assembly, through its Fifth Committee (Administrative and Budgetary) approves and oversees the peacekeeping budget. This includes how specific field operations are funded and equipped, based on detailed submissions provided to it by the UN Secretary-General.

Q Source: Frequently in news: https://peacekeeping.un.org/en/mandates-and-legal-basis-peacekeeping

2 Improved Samba Masuri (ISM) variety rice with low Glycemic Index (Gl) has been developed by the Centre for Cellular and Molecular Biology (CCMB) in association with Indian Institute of Rice Research (IIRR). What are the advantages of this variety of rice? www.insightsias.com 1 TEST - 15(MOCK TEST - 12)

1. It consumes much lesser water despite being coarser grained than non-GM rice. 2. It is resistant to bacteria blight. 3. Rice with low Gl is considered suitable for people with Diabetes.

Select the correct answer using the codes below. 1 onlyA. B. 2 and 3 only C. 1 and 3 only D. 1, 2 and 3

Your Answer : Correct Answer : B

Answer Justification :

Justification: The Hyderabad based Centre for Cellular and Molecular Biology (CCMB) in association with Indian Institute of Rice Research (IIRR) has developed Improved Samba Masuri (ISM) variety rice with low Glycemic Index (Gl).

Rice with low Gl is considered suitable for people with diabetics. Consumption of food with low Gl results in slow release of glucose into the bloodstream reducing the ill-effects of diabetes.

ISM has lowest Gl against the normal in several rice varieties. It also has shown better yield and fine grain type, enhancing its market potential and profit for farmers. It is also resistant to bacteria blight.

The new variety is not a transgenic one, thus improving yield of up to 40%.

The ISM has been grown in 1.3 lakh hectares in 7 states including Telangana, Andhra Pradesh, Tamil Nadu, Uttar Pradesh and Chhattisgarh.

Q Source: http://www.thehindu.com/news/cities/Hyderabad/ccmb-iirr-tie-up-for-low-gi-rice/article20607229.ec e

3 The Ilbert Bill Controversy is said to be a high watermark in the history of Indian National Movement. This is because it invoked issues of 1. Racial discrimination between Indian and Europeans 2. Security lapse on the Indian borders, especially the North-Western frontier 3. Suppression of Indian media houses and their nationalization by the Government

Select the correct answer using the codes below. 1 onlyA. www.insightsias.com 2 TEST - 15(MOCK TEST - 12)

B. 1 and 2 only C. 2 and 3 only D. 1 and 3 only

Your Answer : Correct Answer : A

Answer Justification :

Background & Justification: Lord Ripon wanted to remove two kinds of law that had been prevalent in .

According to the system of law, a European could be tried only by a European Judge or a European Magistrate.

The disqualification was unjust and it was sought to cast a needless discredit and dishonour upon the Indian-born members of the judiciary.

C.P. Ilbert, Law Member, introduced a bill in 1883 to abolish this discrimination in judiciary. But Europeans opposed this Bill strongly.

They even raised a fund of one lakh fifty thousand rupees and established an organisation called the Defence Association.

They also suggested that it was better to end the English rule in India than to allow the English to be subjected to the Indian Judges and Magistrates.

The press in England joined the issue. Hence, Ripon amended the bill to satisfy the English in India and England.

Learning: The Ilbert Bill controversy helped the cause of Indian .

Ripon was totally disillusioned and heartbroken and he tendered his resignation and left for England. The immediate result of this awakening of India was the birth of the in 1885, the very next year of Ripon’s departure.

Q Source: Page 119: 12th TN History Textbook

4 With reference to the Fisheries sector, consider the following statements. 1. The sector contributes nearly 1% of the GDP of India. 2. It accounts for around 10% of the total export value of the country.

Select the correct answer using the codes below. www.insightsias.com 3 TEST - 15(MOCK TEST - 12)

1 onlyA. 2 onlyB. C. Both 1 and 2 NoneD.

Your Answer : C Correct Answer : C

Answer Justification :

Justification: Statement 1: Constituting about 6.3% of the global fish production, the sector contributes to 1.1% of the GDP and 5.15% of the agricultural GDP. The total fish production of 10.07 million metric tonnes presently has nearly 65% contribution from the inland sector and nearly the same from culture fisheries.

Statement 2: Fish and fish products have presently emerged as the largest group in agricultural exports of India, with over 10.5 lakh tonnes in terms of quantity and over Rs. 33,000 crores in value.

This accounts for around 10% of the total exports of the country and nearly 20% of the agricultural exports. More than 50 different types of fish and shellfish products are exported to 75 countries around the world

Learning: With diverse resources ranging from deep seas to lakes in the mountains and more than 10% of the global biodiversity in terms of fish and shellfish species, the country has shown continuous and sustained increments in fish production since independence.

With high growth rates, the different facets of marine fisheries, coastal aquaculture, inland fisheries, freshwater aquaculture, coldwater fisheries to food, health, economy, exports, employment and tourism of the country.

Q Source: http://www.thehindu.com/news/national/tamil-nadu/bjp-observes-world-fisheries-day/article2063147 0.ece

http://nfdb.gov.in/about-indian-fisheries.htm

5 Being at the confluence of the Western Ghats and the Eastern Ghats, the sanctuary is home to eco- systems that are unique to both the mountain ranges. The site is also a declared a tiger reserve. It is

A. Kanger Ghati Wildlife Sanctuary www.insightsias.com 4 TEST - 15(MOCK TEST - 12)

B. Gugamal Sanctuary C. Biligiriranganatha Swamy Temple Wildlife Sanctuary D. Kudremukh Sanctuary

Your Answer : C Correct Answer : C

Answer Justification :

Justification: The hills are located at the easternmost edge of the Western Ghats and support diverse flora and fauna in view of the various habitat types supported.

The sanctuary derives its name Biligiri (white hill in Kannada) from the white rock face that constitutes the major hill crowned with the temple of Lord Rangaswamy or from the white mist and the silver clouds that cover these hills for a greater part of the year.

The hills are in the Yelandur, Kollegal and Chamarajanagar talukas of Chamarajanagar District of Karnataka. They are contiguous with hills in Sathyamangalam Wildlife Sanctuary.

Learning: Nearby, bannerghatta national park is part of a wildlife corridor for elephants which connects the Biligiriranga Hills and the Sathyamangalam forest.

Bannerghatta is the first biological park in India to have a fenced forested elephant sanctuary where elephants can roam around freely without chains.

www.insightsias.com 5 TEST - 15(MOCK TEST - 12)

Image: Biligiriranganatha Swamy Temple

Q Source: https://en.wikipedia.org/wiki/Biligiriranga_Hills

6 Consider the following about Shuddhi movement, in British India. 1. It was a socio-political movement aimed at reducing the conversions of Hindus to Islam and Christianity. 2. It aimed at abolishing the practise of untouchability by converting outcasts from other religions to .

Select the correct answer using the codes below. 1 onlyA. 2 onlyB. C. Both 1 and 2 NoneD.

Your Answer : C Correct Answer : C

Answer Justification :

Justification: Derived from ancient rite of shuddhikaran, or purification, it was started by the Arya Samaj, and its founder Swami Dayanand Saraswati and his followers like Swami Shraddhanand, who also worked on the Sangathan consolidation aspect of Hinduism, in North India, especially Punjab in early 1900s, though it gradually spread across India.

Shuddhi had a social reform agenda behind its belligerent rationale and was aimed at abolishing the practise of untouchability by converting outcasts from other religions to Hinduism and integrating them into the mainstream community by elevating their position, and instilling self-confidence and self-determination in them.

The movement strove to reduce the conversions of Hindus to Islam and Christianity, which were underway at the time.

Q Source: Page 127: 12th TN History Textbook

7 The 'Consumer Welfare Fund', as provided under the Central Goods and Services (GST) Act, was created to 1. Collect all the fines and proceeds from National and State Consumer Dispute Protection Forums 2. Pass the benefits of tax reduction to consumers, if it cannot be directly passed by companies to consumers 3. Compensate the states financially on the account of reduced taxes

Select the correct answer using the codes below. 1 onlyA. B. 2 and 3 only 2 onlyC. www.insightsias.com 6 TEST - 15(MOCK TEST - 12)

D. 1 and 3 only

Your Answer : C Correct Answer : C

Answer Justification :

Justification: The Union Cabinet approved setting up of National Anti-Profiteering Authority (NAA), an apex body with an GST overarching mandate under GST regime so as to ensure the benefit of tax reaches consumers.

The NAA is responsible for applying anti-profiteering measures in event of reduction GST rate on supply of goods or services or benefit of input tax credit is not passed on to recipients by way of commensurate reduction in prices.

If NAA finds that company has not passed on benefits of tax reduction, can direct entity to pass on benefits to consumers along with interest from the date of collection of the higher amount till date of return of such amount.

If the beneficiary cannot be identified, NAA can ask company to transfer amount to the 'Consumer Welfare Fund', as provided under Section 57 of CGST Act.

Q Source: http://pib.nic.in/newsite/PrintRelease.aspx?relid=173563

8 Hunter Commission was appointed in 1882 to

A. Recommend modifications in the Indian Councils Act 1861 for more responsible governance of India B. Review the working of the educational system in India C. Consolidate sunset and sunrise laws related to the municif and panchayat in India D. Assess the secular credentials of the British Government

Your Answer : B Correct Answer : B

Answer Justification :

Background & Learning: Like Lord William Bentinck, Lord Ripon was a champion of education of the Indians. Ripon wanted to review the working of the educational system on the basis of the recommendations of the Wood’s Despatch.

For further improvement of the system Ripon appointed a Commission in 1882 under the chairmanship of Sir William Hunter. The Commission came to be known as the Hunter Commission.

The Commission recommended for the expansion and improvement of the elementary education of the masses. www.insightsias.com 7 TEST - 15(MOCK TEST - 12)

The Commission suggested two channels for the secondary education-one was literary education leading up to the Entrance Examination of the university and the other preparing the students for a vocational career.

The Commission noted the poor status of women education. It encouraged the local bodies in the villages and towns to manage the elementary education.

This had resulted in the extraordinary rise in the number of educational institutions in India.

Q Source: Page 118: 12th TN History Textbook

9 A National Park with Shola forests can be most likely found in which of the following parts of India?

A. Eastern Ghats B. Wetlands near Gulf of Mannar C. Lakshadweep Islands D. Western Ghats

Your Answer : D Correct Answer : D

Answer Justification :

Learning: Sholas are the local name for patches of stunted tropical montane forest found in valleys amid rolling grassland in the higher montane regions of South India.

These patches of shola forest are found mainly in the valleys and are usually separated from one another by undulating montane grassland. The shola and grassland together form the shola-grassland complex or mosaic.

Shola forests are found in the higher altitude hill regions of the Nilgiris, district, the Western Ghats and associated ranges in the states of Karnataka, Kerala and Tamil Nadu.

Although generally said to occur above 2000 meters above sea level, shola forests can be found at 1600 meters elevation in many hill ranges (e.g. Biligiriranga Hills).

Q Source: Parks with Prominent features: https://en.wikipedia.org/wiki/List_of_national_parks_of_India

10 In the context of British India, which among the following mainly contributed to the Vellore Mutiny? 1. New import and export policy of spices near Malabar Coast 2. Annexation of princely states www.insightsias.com 8 TEST - 15(MOCK TEST - 12)

3. Imposition of taxes on locals 4. Introduction of new army regulations

Select the correct answer using the codes below. A. 1 and 2 only B. 2, 3 and 4 only 4 onlyC. D. 1, 2 and 3 only

Your Answer : Correct Answer : C

Answer Justification :

Justification: The Vellore Mutiny was a sepoy mutiny and the causes were rooted in the conditions of the Indian army contingents.

Several causes are attributed to the Vellore Mutiny. Indian sepoys had to experience numerous difficulties when they went to serve in the Company’s army.

The sepoys were forced to serve under the Company since their earlier patrons (the native chieftains) were all disappearing from the scene.

The strict discipline, practice, new weapons, new methods and uniforms were all new to the sepoys.

The sepoys were asked to shave the chin and to trim the moustache. The sepoys felt that these were designed to insult them and their religious and social traditions. There was also a popular belief that this was the beginning of a process by which all of them would be converted to Christianity.

The English treated the Indian sepoys as their inferior. There was the racial prejudice. This was the psychological base for the sepoy mutinies in India during the Company’s rule.

The Vellore uprising was preceded by a series of protests by the Indian troops.

Q Source: Page 97: 12th TN History Textbook

11 With reference to the Joint Polar Satellite System-I (JPSS-I), consider the following statements. 1. It is designed for remote sensing operations and create a Geographical Informational System (GI) based infrared map of the earth. 2. It is a joint venture between NASA and ISRO. www.insightsias.com 9 TEST - 15(MOCK TEST - 12)

3. It will improve recognition of climate patterns that influence weather.

Select the correct answer using the codes below. A. 1 and 2 only B. 2 and 3 only 3 onlyC. D. 1, 2 and 3 only

Your Answer : C Correct Answer : C

Answer Justification :

Justification: Statement 1 and 3: JPSS-I is first multi-day weather forecasts satellite in NOAA's series of four, next-generation operational environmental satellites representing major advancements in observations used for severe weather prediction and environmental monitoring. It is designed to monitor weather around world and help improve forecasts.

Statement 2: JPSS-I is a joint venture between NASA and National Oceanic and Atmospheric Administration (NOAA) of USA.

Learning: It is highly advanced polar-orbiting satellite that will orbit Earth 14 times each day from one pole to other at 824 kms above planet, providing scientists full global coverage twice a day.

JPSS-I carries a suite of advanced instruments designed to take global measurements off atmospheric, land and sea conditions from sea surface temperatures, volcanic ash, hurricane intensity and many more.

Q Source: https://www.nasa.gov/feature/jpss-1-to-provide-more-accurate-environmental-forecasts/

12 With reference to the Partition of Bengal, 1905, consider the following statements. 1. It was done when Lord Dalhousie was the Governor General of British India. 2. The incident gave birth to the Swadeshi and Rithala movement. 3. New provinces of East Bengal and Assam were created after the partition of Bengal.

Select the correct answer using the codes below. A. 1 and 2 only 3 onlyB. C. 2 and 3 only 1 onlyD.

Your Answer : Correct Answer : B

Answer Justification :

Justification: The Partition of Bengal into two provinces was effected in July 1905.

www.insightsias.com 10 TEST - 15(MOCK TEST - 12)

The new province of Eastern Bengal and Assam included the whole of Assam and the Dacca, Rajshahi and Chittagong divisions of Bengal with headquarters at Dacca.

Though Curzon justified his action on administrative lines, partition divided the Hindus and Muslims in Bengal. This led to the anti-partition agitation all over the country.

This had also intensified the National Movement leading to the .

Statement 2: Rithala is a place in , the movement or the riotsin Rithala that happened between 1911-1932 cannot be related to the Swadeshi movement.

Learning: In 1874, Bengal had become too large unit to be administered as a single unit. The government wanted to solve the problem permanently.

It was thought that Assam should be extended and it should include some eastern districts of Bengal.

Congress instead suggested that the Government should separate the Non-bengali speaking areas ( or Orissa) rather than splitting the Bengali speaking area.

This was because the new Western Bengal made the Bengali people minority in their own country because now the population of Hindi and Oriya speaking people was 37 Million while the population of Bengali Speaking people was only 17 Million.

Q Source: Page 118: 12th TN History Textbook

13 Second-line drugs, as against First-line drugs are used in the case of

A. Administered Price Mechanism (APM) B. Chronic case of depression C. Drug resistance diseases D. Dormant disease

Your Answer : C Correct Answer : C

Answer Justification :

Learning: Let us take the example of TB.

Almost all of the antibiotics that can be used to treat TB work when the bacteria are actively dividing.

TB can be treated effectively by using first line drugs (FLD) isoniazid (INH), rifampin (RIF), pyrazinamide (PZA), ethambutol (EMB) and streptomycin (SM). www.insightsias.com 11 TEST - 15(MOCK TEST - 12)

However, this first line therapy often fails to cure TB for several reasons.

Relapse and the spread of the disease contribute to the emergence of drug resistant bacteria.

The emergence of multidrug resistant TB (MDR-TB), i.e. which is resistant to at least isoniazid (INH) and rifampicin (RIF), is of great concern, because it requires the use of second-line drugs that are difficult to procure and are much more toxic and expensive than FLDs.

Therefore, the detection and treatment of drug susceptible or single drug resistant TB is an important strategy for preventing the emergence of MDR-TB.

Q Source: Additional Research: http://pib.nic.in/newsite/PrintRelease.aspx?relid=173607

14 The Climate and Development Knowledge Network (CDKN) is a/an

A. Wing of the United Nations Environment Programme (UNEP) B. Alliance of private organisations and think tanks that links developing countries with experts on climate change C. Inter-Governmental organization established at the Kigali meet of the Conference of Parties (COP) to the UNFCCC D. Non-governmental organization established by the IUCN

Your Answer : B Correct Answer : B

Answer Justification :

Learning: CDKN is run by an alliance of organisations, headed by PricewaterhouseCoopers and including the Overseas Development Institute, a think tank on development and humanitarian issues.

It links developing countries with experts on climate change. Funded by the UK's Department for International Development and the Directorate-General for International Cooperation, its aim is to help developing nations adapt to the consequences of climate change and build capacity for a low- carbon economy.

CDKN does this by combining research, advisory services and knowledge management in support of locally owned and managed policy processes.

Q Source: https://cdkn.org/?loclang=en_gb

http://www.thehindu.com/sci-tech/health/how-ahmedabad-beat-the-heat/article17759591.ece

https://timesofindia.indiatimes.com/city/dehradun/1-budget-allocation-of-each-govt-dept-to-climate-c hange-adaptation/articleshow/55586159.cms www.insightsias.com 12 TEST - 15(MOCK TEST - 12)

15 Monsoon breaks in India can be attributed to 1. Appearance of cold currents on the Western flank of the Indian Peninsula 2. Occurrence of Western Tropical Cyclones coming from the Arabian Sea

Which of the above is/are correct? 1 onlyA. 2 onlyB. C. Both 1 and 2 NoneD.

Your Answer : Correct Answer : D

Answer Justification :

Justification: During the south-west monsoon period after heavy rains for a few days, if rain fails to occur for one or more weeks, it is known as break in the monsoon.

Statement 1: These breaks in rainfall are related to the cyclonic depressions mainly formed at the head of the Bay of Bengal, and their crossing into the mainland. Besides the frequency and intensity of these depressions, the passage followed by them determines the spatial distribution of rainfall.

Moreover, these breaks in the different regions are due to different reasons:

In northern India rains are likely to fail if the rain-bearing storms are not very frequent along the monsoon trough or the ITCZ over this region.

Over the west coast the dry spells are associated with days when winds blow parallel to the coast. So, 1 is correct.

Statement 2: Western Tropical Cyclones originate from the Mediterranean Sea, and they usually withdraw with the onset of the Monsoon.

Q Source: Revision: Page 49: 11th NCERT: India Physical Geography

16 With reference to Rural Electrification Corporation Limited (REC), consider the following statements. 1. It is a Navratna company which finances and promotes rural electrification projects across India. 2. It provides consultancy and technical appraisal support for projects in the role of nodal agency for Government schemes relating to rural power.

Select the correct answer using the codes below. 1 onlyA. 2 onlyB. C. Both 1 and 2 NoneD. www.insightsias.com 13 TEST - 15(MOCK TEST - 12)

Your Answer : Correct Answer : C

Answer Justification :

Justification: Statement 1: REC came into being in 1969. REC is a Navratna Company functioning under the purview of the Ministry of Power. The company is listed on both National Stock Exchange and Bombay Stock Exchange.

The company provides loans to Central/ State Sector Power Utilities, State Electricity Boards, Rural Electric Cooperatives, NGOs and Private Power Developers.

Statement 2: It also provides consultancy, project monitoring and financial/ technical appraisal support for projects, also in the role of nodal agency for Government of India schemes or projects.

The company sanctions loan as a sole lender or co-lender or in consortium with or without the status of lead financer.

REC also offers loan products for financing Renewable Energy projects.

Q Source: Additional Research: http://pib.nic.in/newsite/PrintRelease.aspx?relid=171101

17 With reference to Indira Prize for Peace, Disarmament and Development, consider the following statements. 1. The award is given by the President of India on the recommendation of Union Cabinet headed by the . 2. It can be awarded to both national and international citizens, bodies or organizations.

Select the correct answer using the codes below. 1 onlyA. 2 onlyB. C. Both 1 and 2 NoneD.

www.insightsias.com 14 TEST - 15(MOCK TEST - 12)

Your Answer : Correct Answer : B

Answer Justification :

Justification: Former Prime Minister Manmohan Singh was selected for this prize recently.

He was unanimously chosen for the award by an international jury chaired by former President Pranab Mukherjee.

The prestigious award is accorded annually by Indira Gandhi Trust since 1986. It has been named after former prime minister Indira Gandhi.

It is awarded to individuals or organizations in recognition of their creative efforts towards promoting international peace, development.

It is also bestowed upon them for creating new international economic order and ensuring that scientific discoveries are used for the larger good of humanity and enlarging the scope of freedom.

The award consists of monetary award of 25 lakh and a citation. Some of its earlier recipients are UNICEF (1989), Rajiv Gandhi (1 991), M S Swaminathan (1999), Kofi Annan (2003), Angel Merkel (2013), Indian Space Research Organization (ISRO) (2014) and UN High Commission for Refugees (2015).

Q Source: http://www.thehindu.com/news/national/manmohan-wins-indira-gandhi-prize/article20552490.ece

18 Lord Canning served as Governor General of India from 1856 to 1862, but also served as the First Viceroy of India during that period. How can this be explained?

A. He was a former Secretary of State who was allowed to hold dual positions of power in British India. B. The Government of India Act, 1858, was passed in his tenure. C. He passed legislations with the help of the Central council to assign the position of Viceroy to the Governor General of India. D. Canning was given the title of Viceroy for the exemplary administrative work done under his tenure in India.

Your Answer : Correct Answer : B

Answer Justification : www.insightsias.com 15 TEST - 15(MOCK TEST - 12)

Justification: Lord Canning served as Governor General of India from 1856 to 1862. During his tenure, the Government of India Act, 1858 was passed which created the office of Viceroy to be held by the same person who was Governor General of India. Thus, Lord Canning also served as first Viceroy of India.

After the 1857 Revolt, the responsibility of ruling India was directly assumed by the British Crown.

The Government of India Act of 1858 and the Queen’s Proclamation in the same year signify this change in the Indian administration.

The Queen’s Proclamation remained the basis of the British policy in India for more than 60 years.

Q Source: Page 97: 12th TN History Textbook

19 The largest Marine Protected Area in India is

A. Marine National Park, Andaman B. Gulf of Kutch Marine National Park C. Garimatha Marine Sanctuary D. Chilka Marine Sanctuary

Your Answer : C Correct Answer : C

Answer Justification :

Justification: Option A: Most of the coral reefs in the park are fringing reefs. The type and composition of vegetation varies from island to island. Most notably there you can see a difference between the tourist islands which suffer more anthropocentric change and others which do not. Twin Islands are an important breeding ground for turtles within the park.

Option B: Marine National Park of Gulf of Kutchh is a fragile ecosystem. In recent years, biodiversity of marine park has been under threat on several scores like extraction of corals and sands by cement industries, increased turbidity of water, oil refineries, chemical industries and mechanised fishing boats

Option C: Located in Odisha and a very popular tourist attraction, extending from Dhamra River mouth in the north to Brahmani river mouth in the south, it is the one of world's most important nesting beach for olive ridley turtles.

www.insightsias.com 16 TEST - 15(MOCK TEST - 12)

Image: Nesting Site of Olive Ridley Sea turtles

Q Source: http://www.wiienvis.nic.in/Database/MPA_8098.aspx

20 With reference to Raja Rammohan Roy, consider the following statements. 1. Atmiya Sabha was founded by Debendranath Tagore which was later re-established and renamed as Brahmo Samaj by Rammohan Roy. 2. Brahmo Samaj preached polytheism. 3. He helped William Bentinck to declare the practice of Sati a punishable offence. 4. He did not favour inter-caste marriages.

Select the correct answer using the codes below. www.insightsias.com 17 TEST - 15(MOCK TEST - 12)

A. 1 and 3 only 3 onlyB. C. 2 and 4 only D. 1, 2, 3 and 4

Your Answer : Correct Answer : B

Answer Justification :

Justification: Statement 1 and 2: 1815, he established the Atmiya Sabha. The work of the Atmiya Sabha was carried on by Maharishi Debendranath Tagore (father of Rabindranath Tagore), who renamed it as Brahmo Samaj.

Statement 3: Through this organisation, he preached that there is only one God. He combined the teachings of the Upanishads, the Bible and the Koran in developing unity among the people of different religions.

Raj Rammohan Roy is most remembered for helping Lord William Bentinck to declare the practice of Sati a punishable offence in 1829. He also protested against the child marriage and female infanticide. He favored the remarriage of widows, female education and women’s right to property.

Rammohan Roy started the first Bengali weekly Samvad Kaumudi and edited a Persian weekly Mirat-ul-akhbar. He stood for the freedom of the press.

Statement 4: He believed in the equality of mankind. He did not believe in the supremacy of the Brahmin priests. He favoured inter-caste marriages.

Q Source: Page 127: 12th TN History Textbook

21 With reference to UJjwal DISCOM Assurance Yojana (UDAY), consider the following statements. 1. The scheme is optional for the states to join. 2. It aims at reducing interest burden on discoms by allowing the Central Government to take over the bulk of their debt.

Select the correct answer using the codes below. 1 onlyA. 2 onlyB. C. Both 1 and 2 NoneD.

Your Answer : A Correct Answer : A

www.insightsias.com 18 TEST - 15(MOCK TEST - 12)

Answer Justification :

Justification: UDAY Scheme was launched by Union Power Ministry for financial turnaround and revival package for state electricity distribution companies (DISCOMs). It aimed to help to make discoms financially and operationally healthy so they can supply adequate power at affordable rates.

Statement 1: 27 States and 4 UTS have joined UDAY till date because it is not mandatory for them.

Some State/UTs have joined only for operational improvement and shall not undergo financial restructuring and issue of bonds under the scheme.

Statement 2: Under it, state governments were to take over up to 75% of their respective discoms' debt by issuing sovereign bonds to pay back the lenders. Remaining 25% of debt will be issued by discoms in the form of bonds.

Q Source: http://pib.nic.in/newsite/pmreleases.aspx?mincode=52

22 Curzon brought in the Indian Universities Act which

A. Removed social sciences from the curriculum of private universities to curb nationalism B. Did away with the territorial jurisdiction of the Universities C. Brought all the universities in India under the control of the government D. Nationalized all higher educational institutions in India

Your Answer : Correct Answer : C

Answer Justification :

Background & Justification: Curzon took a serious view of the fall in the standard of education and discipline in the educational institutions.

In his view the universities had degenerated into factories for producing political revolutionaries.

To set the educational system in order, he instituted in 1902, a Universities Commission to go into the entire question of university education in the country.

On the basis of the findings and recommendations of the Commission, Curzon brought in the Indian Universities Act of 1904, which brought all the universities in India under the control of the government.

Option A: This, however, did not mean that social sciences stood removed, but that now the government kept a close vigil of these educational institutions.

www.insightsias.com 19 TEST - 15(MOCK TEST - 12)

Learning: This act allowed the Government to appoint a majority of the fellows in a university.

The Governor General was now empowered to decide a University’s territorial limits and also affiliation between the universities and colleges.

Universities were empowered to appoint their own staff including the teaching staff

However, for education and research a grant per year for 5 years was also accepted.

This was the starting of university grants in India that later became a permanent feature in the structure of India education.

Q Source: Page 120: 12th TN History Textbook

23 The National Park is located where the Deccan Plateau meets the Western Ghats. The park is flanked by the Kabini river in the north and the Moyar river in the south. It is?

A. Bandipur National Park B. Anaimudi National Park C. Kabini Forest Reserve D. Eravikulam National Park

Your Answer : A Correct Answer : A

Answer Justification :

Learning: Together with the adjoining Nagarhole National Park, Mudumalai National Park and Wayanad Wildlife Sanctuary, it is part of the Nilgiri Biosphere Reserve

The altitude of the park ranges from over 2,000 ft to close to 4,770 ft. As a result, the park has a variety of biomes including dry deciduous forests, moist deciduous forests and shrublands. Dry deciduous forest is dominant however.

The wide range of habitats help support a diverse range of organisms. The park is flanked by the Kabini river in the north and the Moyar river in the south. The Nugu river runs through the park.

Option C: Kabini Forest Reserve is surrounded by Wayanad Wildlife Sanctuary, Mudumalai National Park, Bandipur National Park and Nagarhole National Park.

Q Source: https://en.wikipedia.org/wiki/Bandipur_National_Park www.insightsias.com 20 TEST - 15(MOCK TEST - 12)

24 Lord Ripon introduced the (First) Factory Act of 1881 to 1. Improve the service condition of the factory workers in India 2. Categorize organized and unorganized industries in India 3. Push duty free mercantile exports from Small manufacturing units

Select the correct answer using the codes below. 1 onlyA. B. 2 and 3 only C. 1 and 3 only D. 1, 2 and 3

Your Answer : Correct Answer : A

Answer Justification :

Justification: The Act banned the appointment of children below the age of seven in factories. It reduced the working hours for children.

It made compulsory for all dangerous machines in the factories to be properly fenced to ensure security to the workers.

Following this act, a Factory Commission was appointed in 1885.

There was another Factories Act in 1891, and a Royal Commission on Labor was appointed in 1892.

The result of these enactments was the limitation on the factory working hours. This was an answer of the Government to the pathetic conditions of the workers in the factory, wherein, only when a laborer exhausted, new laborer was to take his / her place.

Q Source: Page 118: 12th TN History Textbook

25 Tuberculosis (TB) Detection in India is a major problem and many cases of TB go undetected with the risk of a contagion. In this context, Microbiologically confirmed TB case refers to

A. Any clinically diagnosed case of TB involving organs other than the lungs B. A TB patient who has been diagnosed with active TB by a clinician on the basis of X-ray abnormalities C. a case of patient who has contracted both pulmonary and extrapulmonary TB D. A presumptive TB patient with biological specimen positive for Mycobacterium tuberculosis

Your Answer : www.insightsias.com 21 TEST - 15(MOCK TEST - 12)

Correct Answer : D

Answer Justification :

Justification: There are mainly two types of TB diagnosis:

Clinically diagnosed TB case refers to a presumptive TB patient who is not microbiologically confirmed, but has been diagnosed with active TB by a clinician on the basis of X-ray abnormalities, histopathology or clinical signs with a decision to treat the patient with a full course of Anti-TB treatment.

Microbiologically confirmed TB case is of a presumptive TB patient with biological specimen positive for acid fast bacilli, or positive for Mycobacterium tuberculosis on culture, or positive for tuberculosis through Qualty Assured Rapid Diagnostic molecular test. pulmonary tuberculosis (PTB) refers to any microbiologically confirmed or clinically

Learning: Microbiologically confirmed TB is further classified into two categories:

Pulmonary TB refers to diagnosed case of TB involving the lung parenchyma or the trachea- bronchial tree.

Extra pulmonary tuberculosis (EPTB) refers to any microbiologically confirmed or clinically diagnosed case of TB involving organs other than the lungs such as pleura, lymph nodes, intestine, genitourinary tract, joint and bones, meninges of the brain etc.

Q Source: Additional Research: http://pib.nic.in/newsite/PrintRelease.aspx?relid=173607

https://tbcindia.gov.in/WriteReadData/l892s/8337437943TOG-Chapter%204-Treatment%20of%20TB %20Part%201.pdf

26 What are the advantages of Plastic Organic Light Emitting Diode Screens (POLED) over conventional AMOLED screens and/or IPS LCD Screens? 1. POLED is usually brighter than IPS LCD Screens, more visible during daylight and capable of carrying more colours. 2. POLED provides current to every pixel individually as against AMOLED which provides current to an entire row of LED one at a time. 3. POLED is usually thinner than conventional AMOLED. 4. POLED is less susceptible to cracking and shattering from dropping than conventional AMOLED.

Select the correct answer using the codes below. A. 1 and 2 only B. 3 and 4 only C. 1, 3 and 4 only D. 1, 2, 3 and 4

Your Answer : Correct Answer : B

www.insightsias.com 22 TEST - 15(MOCK TEST - 12)

Answer Justification :

Concept: These are technologies used in the screens of smartphones/tablets or PCs. Each type of screen has its own advantages and trade-offs relating to durability, power consumption, quality of display and flexibility.

OLED (Organic Light Emitting Diodes) is a flat light emitting technology, made by placing a series of organic thin films between two conductors.

When electrical current is applied, a bright light is emitted. OLEDs are emissive display that do not require a backlight and so are thinner and more efficient than LCD displays (which do require a white backlight).

OLEDs are organic because they are made from carbon and hydrogen.

POLED is an OLED panel that uses plastic as a substrate.

OLED display offer improved image quality - better contrast, higher brightness, fuller viewing angle, a wider color range and much faster refresh rates and lower power consumption over an LCD display.

Justification: Statement 2: PMOLED is an OLED control scheme in which each row or line in the display is controlled sequentially and one-by-one. In other words, passive matrix provides current to an entire row of LED one at a time.

On the other hand, AMOLED or active-matrix OLED is a control scheme that uses thin-film transistor backplane to actively maintain a particular pixel state while other pixels are being addressed at the same time. This scheme allows each pixel to be controlled individually.

Statement 1: IPS LCD is essentially brighter than OLED because of the addition of backlighting. This means that a POLED can be difficult to view outdoors or under direct sunlight.

Also, one of the notable disadvantages of POLED is a shorter lifespan than IPS LCD or TN LCD panels.

Statement 3: Typical AMOLED panels such as the Super AMOLED panel from Samsung use glass substrate. Using plastic substrates gives POLED several advantages.

One of the advantages of POLED over conventional AMOLED is thinness. The plastic substrate is considerably half as thin as glass substrates, thus allowing manufacturers to produce thinner display panels that can be fitted into thinner devices.

Another advantage of POLED is flexibility. Take note that LG G Flex was the first smartphone www.insightsias.com 23 TEST - 15(MOCK TEST - 12)

to feature a curved display thanks to its POLED panel.

Plastic is essentially more flexible than glass. It is also worth mentioning that glass is rigid while plastic is easier to form into new shapes.

Statement 4: Plastic OLED is also more durable than typical AMOLED. The flexibility of plastic gives some additional shock absorbance over glass. This durability also means POLED is less susceptible to cracking and shattering from dropping.

Q Source: Introduction of new flagship smartphones that use OLED panels

27 In India or in areas with Indian jurisdiction or where India’s mineral extraction rights prevail, which of the following occupy the largest area in geographical terms?

A. Exclusive Economic Zone B. Continental Shelf C. Brackish waters and Estuaries D. Inland Reservoirs

Your Answer : Correct Answer : A

Answer Justification :

Learning: 1 hectare is equal to 0.01 sq km. So, in this table, the largest is EEZ followed by continental shelf and then Reservoirs.

The resource base matters for resource extraction like fisheries (figures quoted in table below) .

www.insightsias.com 24 TEST - 15(MOCK TEST - 12)

Q Source: http://nfdb.gov.in/about-indian-fisheries.htm

28 Consider the following statements. The Arms Act 1878 1. Prevented any Indian subject to keep arms 2. Provided that Europeans and the Anglo-Indians can keep arms only with appropriate license 3. Mandated that any public gathering of four or more people with arms would be illegal

Which of the above is/are correct? A. 1 and 2 only 2 onlyB. C. 1 and 3 only D. None of the above

Your Answer : C Correct Answer : D

Answer Justification :

Justification: Prior to the Indian First War Of Independence in 1857, there were few gun control laws in India.

But, in the same year as the passing of the Vernacular Press Act, the Arms Act was passed.

www.insightsias.com 25 TEST - 15(MOCK TEST - 12)

This Act prevented the Indians to keep arms without appropriate license. Its violation would be a criminal offence.

The Europeans and the Anglo- Indians were exempted from the operation of these legislations.

The Arms Act, 1959 (amended n 21 was enacted to consolidate and amend the law relating to arms and ammunition in order to curb illegal weapons and violence stemming from them.

Q Source: Page 114: 12th TN History Textbook

29 Consider the following statements. The International Committee of Military Medicine (ICMM) 1. is an international inter-governmental organisation established in after World War II 2. aims to strengthen cooperation between the health services of the armed forces worldwide

Which of the above is/are correct? 1 onlyA. 2 onlyB. C. Both 1 and 2 NoneD.

Your Answer : C Correct Answer : B

Answer Justification :

Justification: The ICMM is international inter-governmental organisation established in 1921 after World War I to strengthen cooperation between the health services of the armed forces worldwide. It currently has 112 nations as members.

ICMM's main objective is to ensure that medical services personnel have means to work together, using similar practices, in operations involving international cooperation.

It works towards achieving this by encouraging activities at which scientific and technical experience is shared. This enables it to pool resources and work experience of military medicine.

Learning: The 42nd World Congress of the International Committee of Military Medicine (ICMM) was held in New Delhi. It was organised by Armed Forces Medical Services (AFMS), Ministry of Defence (MoD).

The theme of this edition of World Congress is "Military Medicine in Transition: Looking www.insightsias.com 26 TEST - 15(MOCK TEST - 12)

Ahead."

The five-day event was being organised for the first time in India. It was the largest medical conference ever organised by AFMS.

It was attended by around 350-400 foreign delegates from 80 countries.

Q Source: http://www.business-standard.com/article/government-press-release/admiral-sunil-lanba-formally-in augurates-42nd-icmm-world-congress-on-military-117112001170_1.html

30 Scientists from Botanical Survey of India (BSI) have discovered a new species of parasitic flowering plant named Gleadovia konyakianorum, that was named in honour of Konyak tribe of Nagas. Consider the following about it. 1. The plant was discovered in dry deciduous forests. 2. The plant is different from others because it has no chlorophyll.

Which of the above is/are correct? 1 onlyA. 2 onlyB. C. Both 1 and 2 NoneD.

Your Answer : B Correct Answer : B

Answer Justification :

Justification: Statement 1: This could have been made out because the Naga tribes do not live in deciduous forests and instead inhabit semi-evergreen and evergreen forests. So, this option could have been eliminated easily.

It was found in a forest at an altitude of 1 ,500-1,600 metres during an exploration near Tobu town of Mon district in eastern Nagaland.

Statement 2: Gleadovia konyakianorum is a holoparasite (complete parasite) plant that derives its entire nutritional requirement from the host plant, which is a Strobilanthes species.

It has no chlorophyll (which helps a plant make its own food using sunlight).

Though it has no chlorophyll, the plant has a vascular system and extracts (its nutrition from the host plant with the help of a haustorium.

Haustorium is specialised structure with which plant parasites attach themselves to host www.insightsias.com 27 TEST - 15(MOCK TEST - 12)

plants and derive nutrition.

Q Source: http://www.thehindu.com/sci-tech/science/scientists-discover-new-parasitic-plant-in-nagaland/article 20551737.ece

31 Malgudi Days was an Indian television series based on the eponymous works of

A. R. K. Narayan B. L. Vaidyanathan C. N.K. Ravi D. Rajshikha Reddy

Your Answer : A Correct Answer : A

Answer Justification :

Learning: The series was directed by Kannada actor and director Shankar Nag. Carnatic musician L. Vaidyanathan composed the score. R. K Narayan's younger brother and acclaimed cartoonist R. K. Laxman was the sketch artist.

The series was made in 1986 by film producer T. S. Narasimhan with Anant Nag as the lead actor.

Actor-playwright Girish Karnad (in Malgudi days) was conferred with 2017 Tata Literature Live Lifetime Achievement Award for his outstanding contribution in field of theatre.

He is also known for his role of Swami's father in the TV series "Malgudi Days". He had served as Director of the Film and Television Institute of India (FTII) and Chairman of Sangeet Natak Akademi, National Academy of Performing Arts.

He is recipient of Jnanpith Award, Padma Bhushan and Padma Shri awards.

Q Source: https://timesofindia.indiatimes.com/life-style/books/features/girish-karnad-wins-lifetime-achievement -award/articleshow/61386816.cms

32 Sangai deer is listed on Schedule-1 of Wildlife (Protection) Act, 1972, and as Endangered on IUCN Red List. Consider the following statements about it. 1. It is the state animal of Manipur and found in its natural habitat only in Manipur. 2. Introduction of Paragrass in Sangai reserves by Government has helped recuperate their population. www.insightsias.com 28 TEST - 15(MOCK TEST - 12)

Select the correct answer using the codes below. 1 onlyA. 2 onlyB. C. Both 1 and 2 NoneD.

Your Answer : A Correct Answer : A

Answer Justification :

Justification: Statement 1: The brow-antlered deer or the dancing deer or Sangai is found in its natural habitat only at Keibul Lamjao National Park over the floating biomass locally called "phumdi" in the south-eastern part of Loktak Lake (Manipur).

Statement 2: Sangai faces threat from steadily degenerating habitat of phumdi as a result of continuous inundation and flooding caused due to artificial reservoir. Water quality of the reservoir is degrading due to pollution and stoppage of nutrient supply. There is also invasion of non-native plants like Paragrass. There has been decrease in area of phumdi

Q Source: Manipur Sangai Festival 2017: http://www.insightsonindia.com/2017/11/22/insights-daily-current-affairs-22-november-2017/

https://www.wwfindia.org/about_wwf/priority_species/threatened_species/brow_antlered_deer/

33 The blue label certification granted by the Marine Stewardship Council essentially depicts that the seafood that is labelled blue is

A. Seafood extracted in a sustainable and eco-friendly way B. Legally extracted from international waters C. The most exotic seafood available in the international market D. Safe for human consumption

Your Answer : A Correct Answer : A

Answer Justification :

Learning: Marine Stewardship Council is an international non-profit organisation.

It grants blue label certification to the fishery across the world and helps create more sustainable seafood market.

MSC certified fisheries minimise their impacts on the whole marine environment to ensure healthy, thriving oceans for the future.

www.insightsias.com 29 TEST - 15(MOCK TEST - 12)

Purchases of MSC labelled seafood create an incentive for more fisheries, retailers and restaurants to produce and sell certified sustainable seafood.

India has special reason this time to celebrate the World Fisheries Day on November 21, as it received international fund for the promotion of sustainable fishing practices.

MSC has awarded a fund for promoting sustainable practices in India’s fisheries sector recently.

Based on this funding, WWF-India and the CMFRI will create fishery management and action plans to ensure the sustainability of both baitfish and tuna fisheries in Lakshadweep, a region which depends on fishing for income and as a food source.

Learning: Besides India, the Council also awarded the fund pre-assessment and action plan development for octopus fisheries in Senegal, improvement action plan for stone crab fishing in Chile, stock assessment for baitfish fisheries in Indonesia, and crayfish project in China.

In all, the MSC has granted a total funds £200,000 to five projects that would aid small scale and Developing World fisheries in achieving sustainability.

The award is part of the MSC’s Global Fisheries Sustainability Fund (GFSF) that was established in 2015 in recognition of the difficulty that these fisheries have in reaching the MSC Standard.

More than 300 fisheries in 34 countries are certified to the MSC’s Standard.

Q Source: http://www.thehindubusinessline.com/news/national/india-receives-international-fund-to-promote-su stainable-fishing-practices/article9966673.ece

34 With reference to the Theosophical Society, consider the following statements: 1. It was founded in Calcutta by Bhikaji Cama. 2. Main aim was to promote the study of ancient religions and philosophies. 3. Central Hindu School was chosen as the parent organization that acted as a catalyst for the growth of this society.

Select the correct answer using the codes below. 2 onlyA. B. 1 and 3 only C. 2 and 3 only D. None of the above

www.insightsias.com 30 TEST - 15(MOCK TEST - 12)

Your Answer : Correct Answer : A

Answer Justification :

Justification: Statement 1: The Theosophical Society was founded in New York (USA) in 1875 by Madam H.P. Blavatsky, a Russian lady, and Henry Steel Olcott, an American colonel.

Statement 2: Their main objectives were to form a universal brotherhood of man without any distinction of race, colour or creed and to promote the study of ancient religions and philosophies. They arrived in India and established their headquarters at Adyar in Madras in 1882.

Statement 3: Later in 1893, Mrs. Annie Besant arrived in India and took over the leadership of the Society after the death of Olcott. Mrs. Annie Besant founded the Central Hindu School along with Madan Mohan Malaviya at Benaras which later developed into the Banaras Hindu University.

Q Source: Page 129: 12th TN History Textbook

35 The Houthis began as a theological movement preaching peace, but now find themselves at the centre of which of these international conflicts?

A. Syrian crisis B. Yemen Crisis C. Israel-Palestine Conflict D. Sudanese Civil War

Your Answer : B Correct Answer : B

Answer Justification :

Learning: It is a Zaidi predominantly Shia-led religious-political movement that emerged from Yemen in the 1990s.

The Houthis took part in the 2011 Yemeni Revolution by participating in street protests and coordinating with other opposition groups, and they did join the National Dialogue Conference in Yemen as part of the Gulf Cooperation Council (GCC) initiative to broker peace following the unrest.

However, the Houthis would later reject the November 2011 GCC deal's provisions, claiming that it did not fundamentally reform governance

In 2014–2015 Houthis took over the government in Sana'a (Yemen).

The Houthis group is considered as a terrorist organization by the governments of Saudi Arabia, the United Arab Emirates and Australia.

A Saudi-led coalition stepped in 2015 and began air strikes on Yemen in an effort to stop the Houthis' advances.

www.insightsias.com 31 TEST - 15(MOCK TEST - 12)

This vitiated the Yemen crisis. You can read more here http://www.insightsonindia.com/2017/11/22/insights-editorial-ripe-intervention/

Q Source: http://www.bbc.com/news/world-middle-east-29319423

36 With reference to the new scheme "Pradhan Mantri Mahila Shakti Kendra (PMMSK)", consider the following statements. 1. It provides for National and State Resource Centre for Women. 2. Community engagement through Student Volunteers is envisioned in the most backward districts. 3. One Stop Centres (OSCs) will be established in selected districts.

Select the correct answer using the codes below. 1 onlyA. B. 2 and 3 only C. 1 and 3 only D. 1, 2 and 3

Your Answer : B Correct Answer : D

Answer Justification :

Justification: Statement 1: It is envisaged to work at various levels. While, National level (domain based knowledge support) and State level (State Resource Centre for Women) structures will provide technical support to the respective government on issues related to women, the District and Block level Centres will provide support to PMMSK.

Statement 2: Community engagement through Student Volunteers is envisioned in 115 most backward districts as part of the PMMSK Block level initiatives. Student volunteers will play an instrumental role in awareness generation regarding various important government schemes/ programmes as well as social issues.

Statement 3: To provide comprehensive support to women affected by violence, One Stop Centres (OSCs) will be established in 150 additional districts during the period.

These one stop Centres will be linked with women helpline and will provide 24 hour emergency and non-emergency response to women affected by violence both in public and private space across the country.

Learning: Also, a unique initiative involving engagement of Manila Police Volunteers (MPVs) on a voluntary basis in States/UTs will. also be done to create public-police interface, which will be expanded to 65 districts covering all States /UTs.

Q Source: AKT/VBA/SH (Release ID :173722)

37 The International Festival of India features a Country of Focus every year. Through this category, prominent films made by the country of focus are screened at IFFI Goa 2017. What is the country of Focus this year? www.insightsias.com 32 TEST - 15(MOCK TEST - 12)

RussiaA. B. Afghanistan CanadaC. D. Germany

Your Answer : C Correct Answer : C

Answer Justification :

Learning: The festival encourages friendly ties between India and other countries through this category, by encouraging delegations from the country, as well as respected individuals from the country’s film industry to visit IFFI.

Canada has been chosen this year as the Focus Country at the 48th International Film Festival of India.

IFFI 2017 is happy to be hosting and presenting a Canadian delegation, in addition to presenting a package of 8 best films from recent Canadian Cinema.

While “Old Stone” was the Canada Focus Opening Film, “Meditation Park” will be the Country Focus - Gala Screening.

Q Source: PIB IFFI 2017/ No. 7 (Release ID :173713)

38 India's Membership for European Bank for Reconstruction & Development (EBRD) will impact India in which of the following ways? 1. India can leverage the technical assistance and sectoral knowledge of the bank for the benefit of development of its private sector. 2. It would enable Indian nationals to get the employment opportunity in the Bank. 3. India’s financial obligations to EBRD would come down as members receive grants from EBRD for their membership. 4. India's investment opportunities would get a boost.

Select the correct answer using the codes below. A. 1, 2 and 4 only B. 1, 3 and 4 only C. 2 and 3 only D. 1, 2, 3 and 4

Your Answer : D Correct Answer : A

Answer Justification :

Justification: Union Cabinet has approved the membership. Necessary steps will be initiated by the Department of Economic Affairs, Ministry of Finance to acquire the membership of the EBRD.

www.insightsias.com 33 TEST - 15(MOCK TEST - 12)

Statement 1 and 4: Membership of EBRD would enhance India's international profile and promote its economic interests. Access to EBRD's Countries of Operation and sector knowledge.

It would increase the scope of cooperation between India and EBRD through co-financing opportunities in manufacturing, services, Information Technology, and Energy.

EBRD's core operations pertain to private sector development in their countries of operation. The membership would help India leverage the technical assistance and sectoral knowledge of the bank for the benefit of development of private sector.

Statement 2: Membership entails this provision.

Statement 3: The minimum initial investment towards the membership of EBRD will be approximately €1 (one) million.

However, this assumption is based on India deciding to buy the minimum number of shares (100) required for obtaining the membership.

If India were to buy a higher number of Bank shares, the financial implications could be higher. In- principle approval of the Cabinet at this stage is being obtained for joining the Bank.

Q Source: AKT/VBA/SH (Release ID :173734)

39 Which of the following bodies decides the Wage Policy for Various rounds of Wage Negotiations for workmen in Central Public Sector Enterprises (CPSEs)?

A. Labour Bureau B. Reserve Bank of India C. Union Cabinet D. National Employment Exchange

Your Answer : C Correct Answer : C

Answer Justification :

Learning: The Union Cabinet recently approved the Wage Policy for the 8th Round of Wage Negotiations for workmen in CPSEs.

Some of the Highlights are:

No budgetary support for any wage increase shall be provided by the Government. The entire financial implication would be borne by the respective CPSEs from their internal resources.

CPSEs must ensure that any increase in wages after negotiations does not result in increase in administered prices of their goods and services.

www.insightsias.com 34 TEST - 15(MOCK TEST - 12)

The wage revision shall be subject to the condition that there shall be no increase in labour cost per physical unit of output.

The management of the concerned CPSEs have to ensure that negotiated scales of pay do not exceed the existing scales of pay of executives/officers and non-unionized supervisors of respective CPSEs.

Q Source: AKT/VBA/SH (Release ID :173724)

40 Introduction of uniform salt tax throughout British India, that later inspired movements like , was associated with?

A. Lord Lytton B. Lord Mayo C. Lord Cunningham D. Lord Dufferin

Your Answer : Correct Answer : A

Answer Justification :

Learning: He also abolished many import duties and supported the Free Trade Policy. This had seriously affected the Indian economic interest.

The system of decentralisation of finance that had begun in the time of

Lord Mayo was continued during the time of Lord Lytton.

The provincial governments were empowered with some control over the expenditure of all provincial matters like land-revenue, excise, stamps, law and justice.

Lytton wanted to encourage the provinces in collecting the revenue and thereby strengthen the financial power and position of the provinces.

In 1878, the Statutory Civil Service was established exclusively for Indians but this was abolished later.

Q Source: Page 115: 12th TN History Textbook

41 With reference to the Global Conference on Cyber Space (GCCS) 2017 being held in India, consider the following statements. www.insightsias.com 35 TEST - 15(MOCK TEST - 12)

1. GCCS was launched with a view to establish internationally agreed ‘rules of the road’ for behaviour in cyberspace. 2. The GCCS is taking place outside OECD nations for the first time.

Select the correct answer using the codes below. 1 onlyA. 2 onlyB. C. Both 1 and 2 NoneD.

Your Answer : Correct Answer : C

Answer Justification :

Justification: It is a prestigious global event where international leaders, policymakers, industry experts, think tanks etc gather to deliberate on issues and challenges for optimally using cyber space.

India is one of the key leaders in the digital and knowledge based economy with over 50% share of the outsourcing market.

India is today the third largest hub for technology start ups, and Indians are recognised for their entrepreneurship spirit globally.

With innovative, technology led programmes such as AADHAAR, MyGov, GeM, Digital Locker, UMANG etc, the new India is the land of technological prowess and transformation.

The theme for the GCCS 2017 is ‘Cyber4All: An Inclusive, Sustainable, Developmental, Safe and Secure Cyberspace’

Q Source: https://gccs2017.in/about

42 Given the importance and potential of medicinal plants, AYUSH Ministry along with certain Stakeholders recently organized a review meeting to examine the draft of National Policy of Medicinal and Aromatic Plants. It was pointed out that most of the Medicinal plants in India are sourced from

OceansA. B. Inland river beds ForestsC. D. Botanical gardens created for that purpose

Your Answer : C www.insightsias.com 36 TEST - 15(MOCK TEST - 12)

Correct Answer : C

Answer Justification :

Justification: More than 80% Medicinal plants are sourced from the forest and this area have the potential of generating lot of employment opportunities.

Also, the fringe areas of rich biodiversity sites are inhabited by the poorest of the poor, whose incomes can be augmented by a comprehensive strategy aimed at capacity building, provision of value addition facilities and structuring an interface between the community and industry.

Medicinal Plant Sector is an ancient as well as a sunrise Sector. India could play a leading role in the global market of herbal product and herbal drugs.

Learning: In order to promote medicinal plants sector, the Government of India has set up National Medicinal Plants Board (NMPB) in 2000.

Currently the board is located in Ministry of AYUSH (Ayurveda, Yoga & Naturopathy, Unani, Siddha & Homoeopathy), Government of India.

The primary mandate of NMPB is to develop an appropriate mechanism for coordination between various ministries/ departments/ organizations in India and implements support policies/programs for overall (conservation, cultivation, trade and export) growth of medicinal plants sector both at the Central /State and International level.

Q Source: http://pib.nic.in/newsite/PrintRelease.aspx?relid=173752

43 The Logistics Sector has been granted Infrastructure status by the Finance Ministry. What are the implications of this move for this sector? 1. It becomes eligible to borrow from India Infrastructure Financing Company Limited (IIFCL). 2. It can now access External Commercial Borrowings (ECB) which is unavailable to non- infrastructure sectors.

Which of the above is/are correct? 1 onlyA. 2 onlyB. C. Both 1 and 2 NoneD.

Your Answer : Correct Answer : A

Answer Justification :

Justification: The need for integrated Logistics sector development has been felt for quite some time in view of the fact that the logistics cost in India is very high compared to developed countries.

High logistics cost reduces the competitiveness of Indian goods both in domestic as well as export market. www.insightsias.com 37 TEST - 15(MOCK TEST - 12)

It will enable the Logistics Sector to avail infrastructure lending at easier terms with enhanced limits, access to larger amounts of funds as External Commercial Borrowings (ECB), access to longer tenor funds from insurance companies and pension funds and be eligible to borrow from India Infrastructure Financing Company Limited (IIFCL).

Q Source: DSM/SBS/KA (Release ID :173674)

44 The Vernacular Press Act, 1878, empowered a Magistrate to 1. Secure an undertaking from the editor of a vernacular newspaper that nothing would be published against the English Government 2. Shut down English presses established by Indians 3. Stop government funding to a media house if it does not adhere to the government policy of expanding the reach of newspapers to rural areas

Select the correct answer using the codes below. A. 1 and 2 only 1 onlyB. C. 2 and 3 only D. 1 and 3 only

Your Answer : Correct Answer : B

Answer Justification :

Justification: This Act empowered a Magistrate to secure an undertaking from the editor, publisher and printer of a vernacular newspaper that nothing would be published against the English Government.

The equipment of the press could be seized if the offence was committed. This Act crushed the freedom of the Indian press.

This was in response to the opposition that had grown with the outset of the Second Anglo-Afghan War (1878–80).

The act excluded English-language publications.

This created adverse public opinion against the British Government.

Q Source: Page 114: 12th TN History Textbook

45 With reference to Private Financing Advisory Network (PFAN), consider the following statements. 1. It is being hosted by UN Industrial Development Organization (UNIDO) and Renewable Energy and Energy Efficiency Partnership (REEEP). 2. It identifies promising clean energy projects at an early stage and provides mentoring for development of a suitable related business plan.

www.insightsias.com 38 TEST - 15(MOCK TEST - 12)

Which of the above is/are correct? 1 onlyA. 2 onlyB. C. Both 1 and 2 NoneD.

Your Answer : Correct Answer : C

Answer Justification :

Justification: PFAN analyses promising clean energy projects at an early stage and provides mentoring for development of a business plan, investment pitch, and growth strategy, significantly enhancing the prospect of financial closure.

Around 360 clean energy projects have been inducted into the PFAN Project Development Pipeline.

These projects employ technologies including biogas, biomass, waste to energy, clean transport, wind, solar, small hydro and energy efficiency solutions.

In May 2017, PFAN was relaunched under a new hosting arrangement with UNIDO and REEEP. Under this new hosting its operations will be scaled up by a factor of two to five by 2020.

Q Source: https://www.unido.org/news/press/clean-energy-and-cli.html

46 The ‘Peace Clause’, often seen in news is related to which of following issues under discussion World Trade Organization (WTO)?

A. Food security of developing nations B. Economic sanctions on Iran by P5+1 nations C. U.S. withdrawal from Trans-Pacific Partnership (TPP) D. Protection of Island nations from onslaught of Climate change related trade policies

Your Answer : A Correct Answer : A

Answer Justification :

Learning: The issue has been discussed several times at our current affairs sections and editorials.

At the upcoming WTO’s Ministerial Conference, India has decided not to agree to severe restrictions on its right to give price subsidies to farmers through the Minimum Support Price www.insightsias.com 39 TEST - 15(MOCK TEST - 12)

(MSP) to procure grains from them for food security purposes.

Food security and protection of low-income and resource-poor farmers are top priority items for India at the WTO meet.

Currently, an interim mechanism called the ‘Peace Clause’ is in place, as per which WTO members had agreed not to challenge developing nations at the WTO Dispute Settlement Mechanism if they breached the cap of the product-specific domestic support (which is 10% of the value of production).

The ‘Peace Clause’ is available to developing nations, including India, till a permanent solution is found to public stockholding for food security purposes.

Q Source: http://www.insightsonindia.com/2017/11/22/insights-daily-current-affairs-22-november-2017/

47 What is notable about the legacy of Lord Ripon? 1. He helped the growth of local bodies like the Municipal Committees in towns and the local boards in taluks and villages. 2. He was responsible for the rendition of Mysore to its Hindu ruler. 3. He introduced a uniform primary education policy throughout British India.

Select the correct answer using the codes below. A. 1 and 2 only 1 onlyB. C. 2 and 3 only D. 1 and 3 only

Your Answer : Correct Answer : A

Answer Justification :

Justification: He devoted himself to task of liberalising the Indian administration.

He repealed the Vernacular Press Act and earned much popularity among Indians.

Ripon believed that self-government is the highest and noblest principles of politics. Therefore, he powers of municipalities were increased.

Their chairmen were to be non-officials. They were entrusted the care of local amenities, sanitation, drainage and water-supply and also primary education. District and taluk boards www.insightsias.com 40 TEST - 15(MOCK TEST - 12)

were created.

It was insisted that the majority of the members of these boards should be elected non- officials. The local bodies were given executive powers with financial resources of their own.

Q Source: Page 117: 12th TN History Textbook

48 The State of the World’s Children report is published by

A. Child Rights and You (CRY) B. United Nations Children’s Fund (UNICEF) C. Oxfam International D. Child Rights Information Network (CRIN)

Your Answer : B Correct Answer : B

Answer Justification :

Learning: This is a lateral explanation, i.e. we try to cover related topics, and not deal with the subject at hand because its obvious.

International Society for Prevention of Child Abuse and Neglect (ISPCAN)

A non-governmental organization seeking to support individuals and organizations working to protect children from abuse and neglect worldwide.

SAFE KIDS Worldwide

It is a global network of organizations dedicated to the prevention of unintentional childhood injury.

The Child Accident Prevention Foundation of Southern Africa (CAPFSA) promotes optimal health and development of all children, via its Childsafe programme.

CAPFSA aims to reduce intentional and unintentional injuries of all severity through research, education, environmental change and recommendations for legislation.

Q Source: http://www.livemint.com/Politics/is2lg1ZmRyRryhVZIjqYeI/India-reelected-as-member-of-UNESCOs- executive-board-offi.html

http://www.who.int/violence_injury_prevention/child/organizations/en/

49 How does Warli art form differ from other tribal art forms? 1. Unlike other tribal art forms, Warli paintings do not employ religious iconography and is a more secular art form. www.insightsias.com 41 TEST - 15(MOCK TEST - 12)

2. Dots and crooked lines are not used in Warli arts, that are frequently used in other tribal art forms. 3. Warli paintings are made mostly by the men, whereas most other tribal paintings are made by women.

Select the correct answer using the codes below. 1 onlyA. B. 2 and 3 only C. 1 and 2 only 2 onlyD.

Your Answer : Correct Answer : A

Answer Justification :

Justification: Warli painting derives its name from a small tribe inhabiting the remote, tribal regions of Maharashtra.

These are decorative paintings on floors and walls of ‘gond’ and ‘kol’ tribes’ homes and places of worship. Trees, birds, men and women collaborate to create a composite whole in a Warli painting.

These paintings are made mostly by the women as part of their routine at auspicious celebrations.

Subjects are drawn with simple and local materials like white colour and rice paste and local vegetable glue on a plain contrasting background, made in a geometric patterns like squares, triangles, and circles.

Dots and crooked lines are the units of these composition. Flora and fauna and people’s day to day life also form a part of the painted.

The paintings are expanded by adding subject after subject in a spiraling manner. The rhythm of the Warli way of life is beautifully captured in simple images.

Learning: Marriage is the most recurring theme of Warli paintings. Many Warli paintings depict Warli Paintings - Marriage Theme Palghat, the marriage god, accompanied by a horse and of course the bride and the groom.

They consider these paintings sacred. Men and women dancing in circles (Tarapa Dance), during various celebrations, is another theme typical to the Warli Paintings.

A musician playing a native instrument is usually found in the middle of such spirals (Tarapa). www.insightsias.com 42 TEST - 15(MOCK TEST - 12)

Flora and fauna are also depicted in these paintings.

Q Source: Revision: NIOS Arts and Culture: On Indian Paintings

50 What is Ocean glider?

A. Dead mass of phumdis and phytoplanktons swayed over by ocean currents B. An autonomous and unmanned underwater vehicle used for ocean science. C. Leftover of oil extractions from mineral exploration that glides over surface ocean currents D. Amphibious aircraft carrier that also serves as a warship

Your Answer : B Correct Answer : B

Answer Justification :

Learning: is an autonomous, unmanned underwater vehicle used for ocean science. Since gliders require little or no human assistance while traveling, these little robots are uniquely suited for collecting data in remote locations, safely and at relatively low cost.

Gliders may be equipped with a wide variety of sensors to monitor temperature, salinity, currents, and other ocean conditions.

This information creates a more complete picture of what is happening in the ocean, as well as trends scientists might not otherwise be able to detect from satellites or large research ships.

Scientists are now experimenting with using gliders to locate populations of spawning fish.

www.insightsias.com 43 TEST - 15(MOCK TEST - 12)

Q Source: https://oceanservice.noaa.gov/facts/ocean-gliders.html

51 Hind or Indian Home Rule is a book written by Mohandas K. Gandhi in 1909. The Gujarati edition was banned by the British on its publication in India but the English edition was not banned because

A. The book was never published in English in India B. British believed that English edition would not affect the majority of Indian population that was native Hindi C. Gandhi revised the English edition cutting down the radical and seditious content D. The then Viceroy favoured the book

Your Answer : Correct Answer : B

Answer Justification :

Justification & Learning: Mohandas Gandhi wrote this book in his native language, Gujarati, while traveling from London to South Africa onboard SS Kildonan Castle in 1909.

In it he expresses his views on Swaraj, modern civilization, mechanisation etc

The Gujarati edition was banned by the British on its publication in India. Gandhi then translated it into English.

The English edition was not banned by the British, who concluded that the book would have little impact on the English-speaking Indians' subservience to the British and British ideas.

Q Source: Additional Research: Page 61: 10th NCERT History

52 The Cornwallis Code is a body of legislation enacted in 1793 by the East India Company to improve the governance of its territories in India. It provided which of the following? 1. The local administration was placed in the hands of the revenue collectors of districts. 2. The land revenue assessment was fixed permanently with zamindars. 3. East India Company’s service personnel be divided into three branches: revenue, judicial, and commercial. 4. Private trade was forbidden to any members of the East India Company.

Select the correct answer using the codes below. A. 2 and 4 only B. 1, 2 and 3 only C. 1, 3 and 4 only D. 2, 3 and 4 only

www.insightsias.com 44 TEST - 15(MOCK TEST - 12)

Your Answer : Correct Answer : B

Answer Justification :

Justification: Statement 2: The code contained significant provisions - governing, policing and judicial and civil administration and Its best known provision was the Permanent Settlement (or the zamindari system enacted in 1793), which established a revenue collection scheme which lasted until the 20th century.

On these the government of British India virtually rested until the Charter Act of 1833.

This “permanent settlement” provided the British with an Indian landed class interested in supporting British authority. The local administration was placed in the hands of the revenue collectors of districts.

Statement 1 and 3: The judiciary was reorganized; there were district judges with magisterial powers responsible to provincial courts in civil cases and to courts of circuit in criminal cases.

The law administered was Hindu and Muslim personal law and a modified Muslim criminal code. The higher ranks of the services were restricted to the British, thus depriving the Indians of any responsible office.

Statement 4: Private trade was forbidden to the members of the first two branches, and they were instead compensated by a new and generous scale of pay. The land revenue assessment (the major source of revenue) was fixed permanently with zamindars, or hereditary revenue collectors.

These native Indians, provided they paid their land taxes punctually, were treated as landowners, but they were deprived of magisterial and police functions, which were discharged by a newly organized government police.

Q Source: Page 259: 12th NCERT History

53 The highest peak in India south of the Himalayas is inside this park. Many perennial streams criss- cross the park. A UNESCO World Heritage Site, it is

A. Silent Valley National Park B. Singalila National Park C. Kudremukh National Park D. Eravikulam National Park

Your Answer : D Correct Answer : D

Answer Justification :

www.insightsias.com 45 TEST - 15(MOCK TEST - 12)

Learning: It is located along the Western Ghats in the Idukki district of Kerala.

It also runs the nearby Mathikettan Shola National Park, Anamudi Shola National Park, Pambadum Shola National Park, Chinnar Wildlife Sanctuary and the Kurinjimala Sanctuary.

It consists of a high rolling hill plateau with a base elevation of about 2,000 m. The terrain consists of high altitude grasslands interspersed with sholas.

Many perennial streams criss-cross the park. They merge to form tributaries of the Periyar river in the west and of the Cauvery River in the east.

Twenty six species of mammals have been recorded in the park including the largest surviving population of Nilgiri tahr

You can read more about the Fauna here https://en.wikipedia.org/wiki/Eravikulam_National_Park#Fauna

Q Source: Protected areas: Southern India

54 The Railway Convention Committee is 1. A standing committee of the Parliament 2. Consists of members of both Lok Sabha and Rajya Sabha 3. Chairman of the Committee is appointed by the Speaker 4. Union Minister of Finance and Minister of Railways have always been ex-officio members of the committee

Select the correct answer using the codes below. A. 1 and 4 only B. 2 and 3 only C. 1, 2 and 3 only D. 1, 2, 3 and 4

Your Answer : Correct Answer : B

Answer Justification :

Justification: It is an ad hoc Committee constituted from time to time on Resolution adopted by Lok Sabha and concurred in by Rajya Sabha. It consists of 18 members.

The 12 members from Lok Sabha nominated by the Speaker and 6 members from Rajya Sabha nominated by the Chairman, Rajya Sabha are associated with the Committee.

www.insightsias.com 46 TEST - 15(MOCK TEST - 12)

The Chairman of the Committee is appointed by the Speaker from amongst the members of the Committee.

The Minister of Finance and the Minister of Railways were hitherto among the members nominated to the Committee. However, in 1989 (9th Lok Sabha), for the first time, the Minister of Finance was not nominated as a Member of the Committee.

After change of Government at the Centre in 1990, the new Railway Minister was not nominated to the Committee and the former Railway Minister continued to be the member of the Committee. So, there is no permanency regarding their presence.

The Committee reviews the rate of dividend which is payable by the Railway Undertaking to the General Revenues as well as other ancillary matters in connection with the Railway Finance vis-a- vis General Finance and make recommendations thereon

Q Source: AKT/VBA/SH (Release ID :173560)

http://www.parliamentofindia.nic.in/ls/intro/railway_convention_committee.htm

55 With reference to British India, consider the following statements. Francis Buchanan 1. was a physician who visited India 2. was a surgeon to one of the Governor-General of India 3. undertook detailed surveys of the areas under the jurisdiction of the East India Company

Select the correct answer using the codes below. 1 onlyA. B. 2 and 3 only C. 1 and 3 only D. 1, 2 and 3

Your Answer : Correct Answer : D

Answer Justification :

Justification: He served in the Bengal Medical Service (from 1794 to 1815).

For a few years he was surgeon to the Governor-General of India, Lord Wellesley.

During his stay in Calcutta (present-day ), he organised a zoo that became the Calcutta Alipore Zoo; he was also in charge of the Botanical Gardens for a short period.

On the request of the Government of Bengal, he undertook detailed surveys of the areas www.insightsias.com 47 TEST - 15(MOCK TEST - 12)

under the jurisdiction of the British East India Company.

In 1815 he fell ill and returned to England. He is often called Buchanan-Hamilton.

Q Source: Page 266: 12th NCERT History

56 According to Ricardian ideas, a landowner should have a claim only to

A. Half of the produce of an agricultural land B. ‘Average rent’ that prevailed at a given time C. Taxable part of the produce D. The produce voluntarily given by the cultivators

Your Answer : Correct Answer : B

Answer Justification :

Learning: As per David Ricardo, an economist of the classical period, when the land yielded more than this “average rent”, the landowner had a surplus that the state needed to tax.

If tax was not levied, cultivators were likely to turn into rentiers, and their surplus income was unlikely to be productively invested in the improvement of the land.

Many British officials in India thought that the history of Bengal confirmed Ricardo’s theory. There the zamindars seemed to have turned into rentiers, leasing out land and living on the rental incomes.

It was therefore necessary, the British officials now felt, to have a different system which led to devising of various land revenue arrangements by the British.

Q Source: Page 277: 12th NCERT History

57 In the middle of the 19th Century, which of these factors pushed cotton production in India that later gave a fillip to the textile Industry? 1. Organization of the Cotton Supply Association in India by the British 2. American Civil War that broke out in 1861 3. Construction of the first railways line in India in 1870s

Select the correct answer using the codes below. A. 1 and 2 only B. 2 and 3 only www.insightsias.com 48 TEST - 15(MOCK TEST - 12)

2 onlyC. D. 1 and 3 only

Your Answer : Correct Answer : C

Answer Justification :

Justification: Statement 1: Before the 1860s, three-fourths of raw cotton imports into Britain came from America. British cotton manufacturers had for long been worried about this dependence on American supplies. 1857 the Cotton Supply Association was founded in Britain, and in 1859 the Manchester Cotton Company was formed. Their objective was “to encourage cotton production in every part of the world suited for its growth”.

Statement 2: When the American Civil War broke out in 1861, a wave of panic spread through cotton circles in Britain. Raw cotton imports from America fell to less than three per cent of the normal: from over 2,000,000 bales (of 400 lbs each) in 1861 to 55,000 bales in 1862.

Frantic messages were sent to India and elsewhere to increase cotton exports to Britain. In Bombay, cotton merchants visited the cotton districts to assess supplies and encourage cultivation.

India was seen as a country that could supply cotton to Lancashire if the American supply dried up. It possessed suitable soil, a climate favourable to cotton cultivation, and cheap labour.

Statement 3: It was first constructed in the 1850s and only later became a medium for India to export cotton to Britain and import manufactured textiles.

Q Source: Page 280: 12th NCERT History

58 Consider the following statements with reference to the: Climate Change Performance Index (CCPI) 2018 1. has been released by Greenpeace 2. evaluates all the parties of the UNFCCC 3. major part of evaluation is based on objective indicators of emissions trend and emissions level 4. India is ranked in the bottom 10 countries

Select the correct answer using the codes below. A. 1 and 4 only 3 onlyB. C. 1, 2 and 3 only D. 2 and 3 only

Your Answer : Correct Answer : B

Answer Justification :

Justification: The report was made public on the sidelines of the UN Climate Change negotiations www.insightsias.com 49 TEST - 15(MOCK TEST - 12)

(COP23) in Bonn.

CCPI is an instrument (released by Germanwatch – a NGO) covering 58 countries and supposed to enhance transparency in international climate politics.

Its aim is to encourage political and social pressure on those countries which have, up to now, failed to take ambitious actions on climate protection as well as to highlight countries with best-practice climate policies.

Statement 3: On the basis of standardised criteria, the index evaluates and compares the climate protection performance of 58 countries that are, together, responsible for more than 90% of global energy-related CO2 emissions.

80% of the evaluation is based on objective indicators of emissions trend and emissions level.

20% of the index results are built upon national and international climate policy assessments by more than 200 experts from the respective countries.

Statement 4: India is ranked 14th, an improvement from its 20th position last year. China, with its high emissions and growing energy use over the past five years, still ranks 41st.

Q Source: http://www.insightsonindia.com/2017/11/16/insights-daily-current-affairs-16-november-2017/

59 The state/UT that does not have any National Park or Conservation Reserve or Community Reserve, but hosts few Wildlife Sanctuaries is

A. West Bengal B. Chandigarh C. Haryana BiharD.

Your Answer : Correct Answer : B

Answer Justification :

Learning:

www.insightsias.com 50 TEST - 15(MOCK TEST - 12)

Q Source: Protected Areas: http://www.moef.nic.in/downloads/public-information/protected-area-network.pdf

60 Consequences of the first world war for India and Indian National movement included: 1. Huge increase in defence expenditure which was financed by war loans and increasing taxes 2. Customs duties were lowered to zero to import cheap British products in India 3. The Rowlatt Act was repealed 4. The Indian industries were rejuvenated as demand for industrial goods during the WW-I increased.

Select the correct answer using the codes below. A. 1 and 4 only B. 2, 3 and 4 only C. 1 and 3 only D. 1, 2 and 4 only

Your Answer : Correct Answer : A

Answer Justification :

Justification: The war created a new economic and political situation.

It led to a huge increase in defence expenditure which was financed by war loans and increasing taxes: customs duties were raised and income tax introduced.

Through the war years prices increased – doubling between 1913 and 1918 – leading to www.insightsias.com 51 TEST - 15(MOCK TEST - 12)

extreme hardship for the common people.

Villages were called upon to supply soldiers, and the forced recruitment in rural areas caused widespread anger.

Then in 1918-19 and 1920-21, crops failed in many parts of India, resulting in acute shortages of food. This was accompanied by an influenza epidemic.

According to the census of 1921, 12 to 13 million people perished as a result of famines and the epidemic.

However, the urban India faced a different situation. The Indian industries were rejuvenated as demand for industrial goods during the WW-I increased. The same industrialists lent support the nationalist by funding and indirect participation.

Learning: The FWW also gave a push to Indian National Movement.

End of the world war ended the myth of invincibility of British Empire in India.

The soldiers that returned after war understood the way British were exploiting the Indian. To fight the British they decided to raise the morale of masses.

This led to the rise of national consciousness and soon Non Cooperation movement was launched.

Formation of USSR also led to the spread of the ideology of socialism in India with the formation of CPI and imparted a socialist tinge to freedom struggle.

Q Source: Page 54: 10th NCERT History

61 In the Words of Gandhiji 1. is physical force 2. Satyagraha is passive resistance 3. Satyagraha is pure soul-force 4. In the use of satyagraha, there is no ill-will whatsoever.

Select the correct answer using the codes below. A. 3 and 4 only B. 2, 3 and 4 only C. 1 and 2 only www.insightsias.com 52 TEST - 15(MOCK TEST - 12)

D. 1, 2, 3 and 4

Your Answer : Correct Answer : A

Answer Justification :

Justification: These are the words of Gandhiji:

‘It is said of “passive resistance” that it is the weapon of the weak, but the power which is the subject of this article can be used only by the strong.

This power is not passive resistance; indeed it calls for intense activity. The movement in South Africa was not passive but active …

‘ Satyagraha is not physical force. A satyagrahi does not inflict pain on the adversary; he does not seek his destruction … In the use of satyagraha, there is no ill-will whatever.

‘ Satyagraha is pure soul-force. Truth is the very substance of the soul. That is why this force is called satyagraha. The soul is informed with knowledge. In it burns the flame of love. … is the supreme dharma …

‘It is certain that India cannot rival Britain or Europe in force of arms. The British worship the war-god and they can all of them become, as they are becoming, bearers of arms.

The hundreds of millions in India can never carry arms. They have made the religion of non- violence their own ...’

Q Source: Page 55: 10th NCERT History

62 Consider the following statements about Diver Detection Sonar (DDS) systems. 1. Purpose of this type of sonar system is to provide detection and classification information on underwater threats 2. These are acoustic location systems employed underwater for the detection of divers and submerged swimmer delivery vehicles (SDVs)

Which of the above is/are correct? 1 onlyA. 2 onlyB. C. Both 1 and 2 NoneD.

www.insightsias.com 53 TEST - 15(MOCK TEST - 12)

Your Answer : Correct Answer : C

Answer Justification :

Justification: Subsurface threats are a difficult problem, because reliable detection is available to date chiefly by use of high-resolution active sonar or trained dolphins or sea lions

The purpose of this type of sonar system is to provide detection, tracking and classification information on underwater threats that could endanger property and lives.

Further, this information is useful only to the extent that it is made available to authorities in time to make possible the desired response to the threat, be it deterrent or defensive action.

Government of India’s ‘Make in India’ Policy; Indian Navy, 2017, has concluded a contract with Tata Power Strategic Engineering Division for supply of Portable Diver Detection Sonar (PDDS) under the ‘Buy and Make (Indian)’ category.

Q Source: http://pib.nic.in/newsite/PrintRelease.aspx?relid=173539

63 A Khilafat Committee was formed in Bombay in 1919 with the main objective to

A. defend the Khalifa’s temporal powers B. call off the Khilafat movement C. decide on whether Muslims should participate in the Non-cooperation movement D. All of the above

Your Answer : D Correct Answer : A

Answer Justification :

Justification: The First World War had ended with the defeat of Ottoman Turkey. And there were rumours that a harsh peace treaty was going to be imposed on the Ottoman emperor – the spiritual head of the Islamic world (the Khalifa).

To defend the Khalifa’s temporal powers, a Khilafat Committee was formed in Bombay in March 1919.

A young generation of Muslim leaders like the brothers Muhammad Ali and Shaukat Ali, began discussing with Mahatma Gandhi about the possibility of a united mass action on the issue.

Gandhiji saw this as an opportunity to bring Muslims under the umbrella of a unified national movement.

www.insightsias.com 54 TEST - 15(MOCK TEST - 12)

At the Calcutta session of the Congress in September 1920, he convinced other leaders of the need to start a non-cooperation movement in support of Khilafat as well as for swaraj.

Q Source: Page 56: 10th NCERT History

64 With reference to District Mineral Foundations (DMFs), consider the following statements. 1. It is a trust set up as a non-profit body, in those districts affected by the mining works 2. It was mandated through the Mines and Minerals (Development & Regulation) Amendment Act, (MMDRA) 2015 3. Every holder of a mining lease in the concerned district shall contribute royalty to the DMF.

Select the correct answer using the codes below. 1 onlyA. B. 2 and 3 only C. 1 and 3 only D. 1, 2 and 3

Your Answer : Correct Answer : D

Answer Justification :

Justification: Statement 1: District Mineral Foundation (DMF) is a trust set up as a non-profit body, in those districts affected by the mining works, to work for the interest and benefit of persons and areas affected by mining related operations. It is funded through the contributions from miners.

Statement 2: The Central Government notified on 17 September 2015, the rates of contribution payable by miners to the DMFs.

In case of all mining leases executed before 12th January, 2015 (the date on which MMDR Amendment Act came into force) miners will have to contribute an amount equal to 30% of the royalty payable by them to the DMFs.

Where mining leases are granted after 12.01.2015, the rate of contribution would be 10% of the royalty payable (Subsequent to the enactment of MMDR Amendment Act, mining leases are given out after auctions; hence, a lower levy).

Thus, every holder of a mining lease or a prospecting licence-cum-mining lease shall, in addition to the royalty, pay to the District Mineral Foundation of the district in which their mining operations are carried on.

Q Source: http://www.insightsonindia.com/2017/11/23/insights-daily-current-affairs-23-november-2017/

www.insightsias.com 55 TEST - 15(MOCK TEST - 12)

http://www.arthapedia.in/index.php?title=District_Mineral_Foundation_(DMF)

65 With reference to Rampa Rebellion of 1922, consider the following statements. 1. It was led by Alluri Sitarama Raju. 2. It took the form of guerilla warfare ending with the British acceding forest rights to the tribals. 3. It was the first organized tribal rebellion.

Select the correct answer using the codes below. A. 1 and 2 only 1 onlyB. C. 2 and 3 only D. 1 and 3 only

Your Answer : Correct Answer : B

Answer Justification :

Justification: Statement 1: The Rampa Rebellion of 1922 was a tribal uprising, led by Alluri Sitarama Raju in Godavari Agency of Madras Presidency, British India. It began in August of 1922 and lasted until the capture and killing of Raju in May 1924.

The Rampa administrative area, situated in the hills of what are now the Godavari districts of Andhra Pradesh,

Statement 2: It broke out in August 1922 and took the form of guerilla warfare, ending in May 1924 with the capture and shooting of Raju

Statement 3: The tribal people had long felt that the legal system favoured zamindars (estate landowners) and merchants, which had also resulted in the earlier Rampa Rebellion of 1879. There were other rebellions as well, such as Santhal rebellion etc.

Learning: Tribals had traditionally been able to support their food requirements through the use, in particular, of the podu system whereby each year some areas of jungle forest were burned to clear land for cultivation.

The British Raj authorities had wanted to improve the economic usefulness of lands in Godavari Agency, an area that was noted for the prevalence of malaria and blackwater fever.

The traditional cultivation methods were greatly hindered when the authorities took control of the forests, mostly for commercial purposes such as produce for building railways and ships, without any regard for the needs of the tribal people.

The conflict later catapulted in the Rampa rebellion of 1922.

Q Source: Additional Research: Page 60: 10th NCERT History

66 Plantation workers of Assam opposed the Inland Emigration Act of 1859 because www.insightsias.com 56 TEST - 15(MOCK TEST - 12)

1. It took away their right to free movement 2. It introduced 'free' emigration and unlicensed recruitment.

Which of the above is/are correct? 1 onlyA. 2 onlyB. C. Both 1 and 2 NoneD.

Your Answer : Correct Answer : A

Answer Justification :

Justification: Statement 1: Plantation workers were not permitted to leave the tea gardens without permission and in fact they were rarely given such permission. When they heard of the Non- Cooperation Movement, thousands of workers defied the authorities, left the plantations and headed home.

They believed that Gandhi Raj was coming and everyone would be given land in their own villages. Hence, in opposition to such acts, many participated in the movement.

Statement 2: This was through a later inland migration act.

The Inland Emigration Act of 1882, which introduced 'free' emigration and unlicensed recruitment, also strengthened the penal contract system. Contemporary accounts testify to the ruthlessness with which such power was exercised by the planters

All this reduced indentured wage labour employed on Assam plantations to forced labour or unfree labour.

Q Source: Page 60: 10th NCERT History

67 The kanungos were the

A. hereditary revenue officers since the time of Mughals. B. law givers as the assistant of the Ulemmas C. judicial magistrates in the village panchayats D. bonded labourers released by the British

Your Answer : C Correct Answer : A

Answer Justification :

Learning: The kanungo comes into prominence in the reign of Akbar, who employed him, as the name implies, to keep the records of the pargana, a revenue sub-division.

www.insightsias.com 57 TEST - 15(MOCK TEST - 12)

He was in fact a registrar of a district appointed to see that the crown received its dues and that the ryot was not oppressed; his duties were responsible and onerous; he had to register the usages of a district, the rate and of its and all regulation relating thereto.

The kanungos' duties also included the keeping of a record of all events, such as the appointments, deaths or removals of zamindars; to preserve the records of the Tumar and Taksim Jama etc.

Q Source: Page 259: 12th NCERT History

68 Figures associated with Permanent settlement 1793 were 1. Cornwallis 2. Kenneth Mckenzie 3. Thomas Munro

Select the correct answer using the codes below. 1 onlyA. B. 2 and 3 only C. 1 and 3 only D. 1 and 2 only

Your Answer : Correct Answer : A

Answer Justification :

Justification: The Permanent Settlement, also known as the Permanent Settlement of Bengal was an agreement between the East India Company and Bengali landlords to fix revenues to be raised from land.

It had far-reaching consequences for both agricultural methods and productivity in the entire British Empire and the political realities of the Indian countryside.

Permanent Settlement introduced first in Bengal and Bihar; and then to South district of Madras and district of Varanasi.

The system eventually spread all over northern India by a series of regulations in 1793. These regulations remained in place until the Charter Act of 1833.

Details will be covered in another question.

Q Source: Page 259: 12th NCERT History

69 In the context of Modern India, what is often referred to as the Fifth Report, was actually a

www.insightsias.com 58 TEST - 15(MOCK TEST - 12)

A. Manifesto of the Indian National Congress B. A report on the East India Company’s administration of India C. Compilation of demands of the moderate leaders D. A review of charter acts produced by the British Parliament

Your Answer : Correct Answer : B

Answer Justification :

Background & Learning: From the time the Company established its rule in Bengal in the mid-1760s, its activities were closely watched and debated in England.

There were many groups in Britain who were opposed to the monopoly that the East India Company had over trade with India and China. These groups wanted a revocation of the Royal Charter that gave the Company this monopoly.

Company officials were widely publicised in the press. The British Parliament passed a series of Acts in the late eighteenth century to regulate and control Company rule in India.

It forced the Company to produce regular reports on the administration of India and appointed committees to enquire into the affairs of the Company.

The Fifth Report was one such report produced by a Select Committee.

It became the basis of intense parliamentary debates on the nature of the East India Company’s rule in India.

Q Source: Page 265: 12th NCERT History

70 The consequences of the Great Depression (1929) for India included 1. Mass production of defence goods in India 2. Collapse of prices of many food grains 3. India became a net importer of precious metals like gold.

Select the correct answer using the codes below. A. 1 and 2 only 2 onlyB. C. 2 and 3 only D. 1 and 3 only

Your Answer : Correct Answer : B www.insightsias.com 59 TEST - 15(MOCK TEST - 12)

Answer Justification :

Background: First World War (FWW) lead to mass production in the US and lowered goods prices. With rising wages in US, consumption rocketed as people could afford more goods.

Demand for consumer goods was fuelled by a boom in housing construction and real estate business. This created the base for prosperity in the US leading to greater investments and incomes.

US started exporting capital to the World and became the largest overseas lender. But since 1929 the world along with US economy slumped into a deep recession.

Justification: Implications for Colonial India

India was connected to global economy by way of imports and exports. Both halved between 1928 and 1934.

Negatives

Rural economy suffered more than urban economy.

As international food grain prices collapsed, they declined in India too. For e.g. wheat prices declined by 50% in the same period.

Peasants who had borrowed in hope of good prices in future, suffered from indebtedness because of falling demand and lower prices.

Even as farm incomes lowered, the British government refused to reduce revenue demand. Peasant indebtedness spiked.

Positives

India became a net exporter of previous metals like gold. Keynes thought that India's gold export will help global economic recovery.

Since urban incomes (salaries) were fixed, they were better off as prices had plunged. Effective demand for goods increased.

Industrial investment also grew as the government extended tariff protection to industries, under the pressure of nationalist opinion.

www.insightsias.com 60 TEST - 15(MOCK TEST - 12)

Q Source: Additional Research: Page 62: 10th NCERT History

71 The Religious Freedom Bill passed by the Rajasthan Assembly in 2008 aimed at 1. Banning forcible religious conversions 2. Promoting religious educations in state run schools and colleges

Which of the above is/are correct? 1 onlyA. 2 onlyB. C. Both 1 and 2 NoneD.

Your Answer : Correct Answer : A

Answer Justification :

Justification: It was recently returned by the Union government as it deviated from the national policy. According to the Union Home Ministry, the Bill was sent back for “further clarifications.”

The legislation seeks to stop conversions through use of “force or allurement or by fraudulent means”.

The Bill defined “conversion” as “renouncing one’s own religion and adopting another” through “fraudulent means” or any other “fraudulent contrivance.”

It also contains a clause for cancellation of registration of organisations held guilty of abetting conversions.

An offence under the Act is cognizable and non-bailable.

Q Source: http://www.thehindu.com/news/national/other-states/rajasthan-conversion-bill-returned-by-centre/ar ticle20461261.ece

72 “” is a list that was first coined in

A. Gandhi’s speech in Banaras Hindu University (BHU) B. A political campaign involving the Indian National Congress led by Gokhale C. The book “My Experiments with Truth” by M.K. Gandhi D. None of the above

Your Answer : Correct Answer : D

Answer Justification :

www.insightsias.com 61 TEST - 15(MOCK TEST - 12)

Justification: It's a common misconception that Gandhi was the one who used it first. He published the same list in his weekly newspaper on October 22, 1925.

But, it was actually uttered first in a sermon delivered in Westminster Abbey on March 20, 1925 by an Anglican priest named Frederick Lewis Donaldson.

He originally referred to it as the "7 Deadly Social Evils".

Gandhi later gave the same list to his grandson, Arun Gandhi, written on a piece of paper on their final day together shortly before his assassination.

Learning: The Seven Sins are:

Wealth without work.

Pleasure without conscience.

Knowledge without character.

Commerce without morality.

Science without humanity.

Religion without sacrifice.

Politics without principle.

Q Source: Page 63: 10th NCERT History

73 The Independence Day Pledge taken on 26 January 1930 affirmed that #0000000 1. If any government deprives a people of these rights and oppresses them, the people have a further right to alter it or to abolish it. 2. The British Government deprived the Indian people of their freedom and exploited them economically, politically, culturally and spiritually.

Which of the above is/are correct? 1 onlyA. 2 onlyB. www.insightsias.com 62 TEST - 15(MOCK TEST - 12)

C. Both 1 and 2 NoneD.

Your Answer : Correct Answer : C

Answer Justification :

Justification: The pledge reads ‘We believe that it is the inalienable right of the Indian people, as of any other people, to have freedom and to enjoy the fruits of their toil and have the necessities of life, so that they may have full opportunities of growth.

We believe also that if any government deprives a people of these rights and oppresses them, the people have a further right to alter it or to abolish it.

The British Government in India has not only deprived the Indian people of their freedom but has based itself on the exploitation of the masses, and has ruined India economically, politically, culturally, and spiritually.

We believe, therefore, that India must sever the British connection and attain or Complete Independence.’

Q Source: Page 63: 10th NCERT History

74 Which of the following characterize Dandi march, consider the following statements. 1. March covering 1000 miles 2. Over 1000 volunteers marching together 3. Spanned over 500 hours

Select the correct answer using the codes below. A. 1 and 2 only 3 onlyB. C. 1 and 3 only D. None of the above

Your Answer : Correct Answer : B

Answer Justification :

Justification: The march was over 240 miles, from Gandhiji’s ashram in Sabarmati to the Gujarati coastal town of Dandi. The volunteers walked for 24 days (576 hours), about 10 miles a day.

Thousands came to hear Mahatma Gandhi wherever he stopped, and he told them what he meant by swaraj and urged them to peacefully defy the British.

On 6 April he reached Dandi, and ceremonially violated the law, manufacturing salt by boiling sea water. www.insightsias.com 63 TEST - 15(MOCK TEST - 12)

Mahatma Gandhi started this march because his demands made to Irwin in a letter were not fulfilled.

Q Source: Page 63: 10th NCERT History

75 A manifesto titled “The Revolutionary”, that was produced as evidence in the Kakori conspiracy case of 1925, was written by

A. Sachindra Nath Sanyal B. W.C. Banerjee C. Feroze Shah Mehta D. Badruddin Tyabji

Your Answer : Correct Answer : A

Answer Justification :

Justification: Use elimination. It is clear that the manifesto was an extremist project, so all options other than A can be rejected because they were all moderates. It was a manifesto of HSRA.

Learning: Hindustan Socialist Republican Association (HSRA) was a revolutionary organisation established in 1928 at Feroz Shah Kotla in New Delhi by Chandrasekhar Azad, Bhagat Singh, Sukhdev Thapar and others.

Sanyal wrote a manifesto for the HRA entitled Revolutionary. This was distributed around large cities of North India in 1924.

It proposed the overthrow of British colonial rule and its replacement with what it termed a "Federal Republic of the United States of India".

In addition, it sought universal suffrage and the socialist-oriented aim of abolition of "all systems which make any kind of exploitation of man by man possible"

Q Source: Page 65: 10th NCERT History

76 The Aligarh Movement was started for

A. For the social and educational advancement of the Muslims in India B. Opposing the establishment of the Indian National Congress C. Creating the All-India Muslim League D. To reform the University system as proposed by the Wood’s Despatch

Your Answer : www.insightsias.com 64 TEST - 15(MOCK TEST - 12)

Correct Answer : A

Answer Justification :

Learning: It was started by Sir Syed Ahmad Khan (1817-98) for the social and educational advancement of the Muslims in India.

He fought against the medieval backwardness and advocated a rational approach towards religion.

In 1866, he started the Mohammadan Educational Conference as a general forum for spreading liberal ideas among the Muslims.

In 1875, he founded a modern school at Aligarh to promote English education among the Muslims. This had later grown into the Mohammadan Anglo Oriental College and then into the Aligarh Muslim University.

Q Source: Page 133: 12th TN History Textbook

77 The famous Buddhist Diamond Sutra is a/an

A. Old Japanese text B. Translation of Abhidhamma pitaka compiled in India C. Recent work of the Gompa monastery, Spiti D. Chinese version of the Sutta pitakas

Your Answer : Correct Answer : A

Answer Justification :

Learning: Buddhist missionaries from China introduced hand-printing technology into Japan around AD 768-770.

The oldest Japanese book, printed in AD 868, is the Buddhist Diamond Sutra, containing six sheets of text and woodcut illustrations.

Diamond Sutra is one of the most influential Mahayana sutras in East Asia, and is a key object of devotion and study in Zen Buddhism.

If interested, you may read more here https://en.wikipedia.org/wiki/Diamond_Sutra

Q Source: Page 154: 10th NCERT History

78 India has sent a proposal to United Nations for declaring the year 2018 as ‘International Year of Millets’. What is the importance of millets in terms of production, use and daily nutrition? 1. It can be used as fodder as well as biofuels. 2. It superior to wheat and rice in higher levels of protein. 3. It does not have amino acids. www.insightsias.com 65 TEST - 15(MOCK TEST - 12)

4. It is high in dietary fibre and antioxidants. 5. It can be grown on dryland farms.

Select the correct answer using the codes below. A. 1, 2, 4 and 5 only B. 2, 3 and 5 only C. 1, 3 and 4 only D. 2 and 5 only

Your Answer : A Correct Answer : A

Answer Justification :

Justification: Millet is a common term to categorize small-seeded grasses that are often termed nutri-cereals or dryland-cereals, and includes sorghum, pearl millet, ragi, small millet, foxtail millet, proso millet, barnyard millet, kodo millet and other millets.

An important staple cereal crop for millions of small holder dryland farmers across sub- saharan Africa and Asia, millets offer nutrition, resilience, income and livelihood for farmers even in difficult times.

They have multiple untapped uses such as food, feed, fodder, biofuels and brewing.

Statement 2, 3 and 4: Nutritionally superior to wheat & rice owing to their higher levels of protein with more balanced amino acid profile, crude fiber & minerals such as Iron, Zinc, and Phosphorous, millets can provide nutritional security and act as a shield against nutritional deficiency, especially among children and women.

The anaemia (iron deficiency), B-complex vitamin deficiency, pellagra (niacin deficiency) can be effectively tackled with intake of less expensive but nutritionally rich food grains like millets.

Millets can also help tackle health challenges such as obesity, diabetes and lifestyle problems as they are gluten free, have a low glycemic index and are high in dietary fibre and antioxidants.

Statement 5: Adapted to low or no purchased inputs and to harsh environment of the semi-arid tropics, they are the backbone for dry land agriculture.

Photo-insensitive & resilient to climate change, millets are hardy, resilient crops that have a low carbon and water footprint, can withstand high temperatures and grow on poor soils with little or no external inputs. www.insightsias.com 66 TEST - 15(MOCK TEST - 12)

In times of climate change they are often the last crop standing and, thus, are a good risk management strategy for resource-poor marginal farmers.

Q Source http://www.insightsonindia.com/2017/11/23/insights-daily-current-affairs-23-november-2017/

79 Deccan Riots of 1875 had its roots in

A. Forest enclosures and new forest laws by the British B. Opposition to the Indian Councils Act 1861 C. Laying down of railways tracks D. Agrarian distress

Your Answer : Correct Answer : D

Answer Justification :

Background & Learning: In 1875, peasants of Maharashtra in some parts of Pune, Satara and Nagar districts revolted against increasing agrarian distress.

The Deccan Riots of 1875 targeted conditions of debt peonage (kamiuti) to moneylenders.

The rioters' specific purpose was to obtain and destroy the bonds, decrees, and other documents in the possession of the moneylenders

he commercialization of agriculture under British land revenue policies burdened small peasants by placing a premium on access to credit to finance productive investments in the land.

Employing capital advanced by European merchants, local moneylenders obtained unlimited title to the property and labor of their debtors; it gave them the "power to utterly ruin and enslave the debtor.

During the 19th century, they used this power to control peasant labour, and not their land, which was of little value without people to work it.

These changes in agriculture undermined the communal traditions which had been the basis of Indian village life.

Aftermath: When the revolt spread in the Deccan, the Government of Bombay was initially unwilling to see it as anything serious. But the Government of India, worried by the memory of www.insightsias.com 67 TEST - 15(MOCK TEST - 12)

1857, pressurised the Government of Bombay to set up a commission of enquiry to investigate into the causes of the riots.

The commission produced a report that was presented to the British Parliament in 1878.

Q Source: Page 284: 12th NCERT History

80 The Round Table Conference was organized to 1. Discuss a future constitution for India 2. Discuss the implementation of a dominion status for India

Which of the above is/are correct? 1 onlyA. 2 onlyB. C. Both 1 and 2 NoneD.

Your Answer : Correct Answer : A

Answer Justification :

Justification: The three Round Table Conferences of 1930–32 were a series of conferences organized by the British Government to discuss constitutional reforms in India.

They were conducted as per the recommendation of Jinnah to Lord Irwin viceroy and his friend Ramsay MacDonald then Prime Minister of Great Britain, and by the report submitted by the Simon Commission in May 1930.

Demands for swaraj, or self-rule, in India had been growing increasingly strong. By the 1930s, many British politicians believed that India needed to move towards dominion status.

However, there were significant disagreements between the Indian and the British political parties that the Conferences would not resolve.

Statement 2: Dominion status was officially promised as a Right only by the Cripps Mission which was much after the Round Table Conference.

Q Source: Page 62: 10th NCERT History

81 With reference to Coastal Economic Zones (CEZs), consider the following statements. 1. CEZs are spatial economic regions comprising of a group of coastal districts or districts with a strong linkage to the ports in that region. 2. Coastal Economic Units (CEUs) will be specific industrial estate projects will house the industrial www.insightsias.com 68 TEST - 15(MOCK TEST - 12)

clusters / projects proposed within the CEZ.

Which of the above is/are correct? 1 onlyA. 2 onlyB. C. Both 1 and 2 NoneD.

Your Answer : Correct Answer : C

Answer Justification :

Justification: CEZs are envisaged to tap synergies with the planned industrial corridor projects.

CEUs will be specific industrial estate projects with a demarcated boundary similar to the DMIC nodes.

Each CEZ will consist of multiple CEUs and more than one industrial cluster can be housed within a CEU.

Within each industrial cluster there can be several manufacturing units.

To accelerate the CEU development process, it is proposed that CEUs be prioritized in locations where land parcels are available in areas close to a deep draught port and with strong potential for manufacturing.

As part of the National Perspective Plan, 14 CEZs have been proposed across all Maritime States & Union Territories and their perspective plans have been prepared. The Master Plans for these CEZs will be developed in a phased manner.

Q Source: http://sagarmala.gov.in/project/port-led-industrialization

https://economictimes.indiatimes.com/news/economy/infrastructure/decks-cleared-for-first-mega-cez -45-companies-may-invest-rs-15k-crore-in-phase-i/articleshow/61663755.cms

82 With reference to Surendranath Banerjee, consider the following statements. 1. He was called the Indian Burke. 2. He founded the Indian Association to agitate for political reforms. 3. He had convened the Servants of India Society which merged with the Indian National Congress. 4. He was the first Indian to become a Member of the British House of Commons.

Select the correct answer using the codes below. www.insightsias.com 69 TEST - 15(MOCK TEST - 12)

A. 1 and 2 only B. 1, 3 and 4 only C. 2 and 4 only D. 1, 2, 3 and 4

Your Answer : Correct Answer : A

Answer Justification :

Justification: Surendranath Banerjee was called the Indian Burke. He firmly opposed the Partition of Bengal.

He founded the Indian Association (1876) to agitate for political reforms. He had convened the Indian National Conference (1883) which merged with the Indian National Congress in 1886.

Dadabhai Naoroji was known as the Grand Old Man of India. He is regarded as India’s unofficial Ambassador in England. He was the first Indian to become a Member of the British House of Commons.

Gopal Krishna Gokhale was regarded as the political guru of Gandhi. In 1905, he founded the Servants of India Society to train Indians to dedicate their lives to the cause of the country.

Q Source: Page 142: 12th TN History Textbook

83 The Young Bengal Movement

A. attacked old traditions and decadent customs B. advocated women’s rights and their education C. organized debates against idol worship D. All of the above

Your Answer : Correct Answer : D

Answer Justification :

Learning: Henry Vivian Derozio was the founder of the Young Bengal Movement. He taught in the Hindu College, Calcutta.

His followers were known as the Derozians and their movement the Young Bengal Movement.

They attacked old traditions and decadent customs. They also advocated women’s rights and their education. www.insightsias.com 70 TEST - 15(MOCK TEST - 12)

They founded associations and organized debates against idol worship, casteism and superstitions.

Q Source: Page 132: 12th TN History Textbook

84 Kanikkarans are the Original tribal Settlers in this protected area. It includes the Indian Ecoregions of South Western Ghats moist deciduous forests, South Western Ghats montane rain forests and Shola. It became part of World Network of Biosphere Reserves in 2016, it is

A. Agasthyamala Biosphere Reserve B. Nilgiri Biosphere Reserve C. Periyar Biosphere Reserve D. Nagarhole Biosphere Reserve

Your Answer : Correct Answer : A

Answer Justification :

Learning: It spans both Kerala and Tamil Nadu regions.

It is the habitat for 2,000 varieties of medicinal plants, of which at least 50 are rare and endangered species.

Rare animals include the tiger, Asian Elephant, and Nilgiri Tahr.

Kanikaran are one of the oldest surviving ancient tribes in the world. Even though it is said their language is a mixture of Tamil and Malayalam dialects, they have their own mother tongue, which is centuries old.

Q Source: Protected areas: Southern India

85 Consider the following statements. Assertion (A): 1857 revolt was largely limited to Northern and Central India. Reason (R): The territories beyond Aurangabad in Southern India were not under British control.

In the context of the above, which of these is correct? A. A is correct, and R is an appropriate explanation of A. B. A is correct, but R is not an appropriate explanation of A. C. A is correct, but R is incorrect. D. A is incorrect, but R is correct.

Your Answer : C Correct Answer : C

Answer Justification :

Justification: The map below shows that it was not so. British control extended almost all across India. www.insightsias.com 71 TEST - 15(MOCK TEST - 12)

Q Source: Page 305: 12th NCERT History

86 Consider the following statements. 1. Nirankari Movement insisted on the worship of the formless God. 2. Namdhari Movement followers started wearing black clothes and performed tantric practices. www.insightsias.com 72 TEST - 15(MOCK TEST - 12)

3. Akalis started a movement to remove the corrupt Mahants from the Sikh gurudwaras, something that turned into a political association later.

Which of the above is/are correct? 1 onlyA. B. 2 and 3 only C. 1 and 3 only D. 1, 2 and 3

Your Answer : Correct Answer : C

Answer Justification :

Justification: Punjab came under the spell of many social reforms.

Baba Dayal Das founded the Nirankari Movement. He insisted the worship of God as nirankar (formless).

The Namdhari Movement was founded by Baba Ram Singh. His followers wore white clothes and gave up meat eating.

The Singh Sabhas started in Lahore and Amritsar in 1870 were aimed at reforming the Sikh society. They helped to set up the Khalsa College at Amritsar in 1892.They also encouraged Gurmukhi and Punjabi literature.

In 1920, the Akalis started a movement to remove the corrupt Mahants (priests) from the Sikh gurudwaras. The British government was forced to make laws on this matter. Later, the Akalis organised themselves into a political party.

Q Source: Page 134: 12th TN History Textbook

87 The title “Socrates of South Asia” is often attributed to

A. Mahatma Gandhi B. Gopalkrishna Gokhale C. Pandit Nehru D. Periyar E.V. Ramaswamy

Your Answer : D Correct Answer : D

Answer Justification :

Learning: Periyar E.V. Ramaswamy was a great social reformer. In 1921, during the anti-liquor campaign he cut down 1000 coconut trees in his own farm.

www.insightsias.com 73 TEST - 15(MOCK TEST - 12)

In 1924, he took an active part in the . The objective of the Satyagraha was to secure for untouchables the right to use a road near a temple at Vaikom in Kerala.

Subsequently in 1925, he started the “Self-Respect Movement”. The aims of the ‘Self -Respect Movement’ were to uplift the Dravidians and to expose the Brahminical tyrany and deceptive methods by which they controlled all spheres of Hindu life.

In 1938 at Tamil Nadu Women’s Conference appreciating the noble service rendered by E.V.R. he was given the title “Periyar”.

UNESCO organisation praised and adorned him with the title “Socrates of South Asia” in 1970.

He attacked the laws of Manu, which he called the basis of the entire Hindu social fabric of caste.

He founded the Tamil journals Kudiarasu, Puratchi and Viduthalai to propagate his ideals.

Q Source: Page 135: 12th TN History Textbook

88 Amongst the following, the earliest to form was

A. The British Indian Association B. The Bombay Association C. The Madras Mahajana Sabha D. Poona Sarvojanik Sabha

Your Answer : Correct Answer : A

Answer Justification :

Justification: These were some of the early Political Associations that paved the way for organized struggle in India:

The British Indian Association – 1851 Bengal

The Bombay Association — 1852 Dadabhai Naoroji

East India Association 1856 London

www.insightsias.com 74 TEST - 15(MOCK TEST - 12)

Madras Native Association 1852

Poona Sarvojanik Sabha—1870

The Madras Mahajana Sabha—1884

We have covered some of them in previous tests, and some will be covered later.

Learning: Bombay Association was started to vent public grievances to the British.

The first organisation in the Madras Presidency to agitate for the rights of Indians was the Madras Native Association which was established by publicist Gazulu Lakshminarasu Chetty in 1849. This organisation did not survive for long and was eventually disbanded.

The members of the Sabha felt the necessity of creating an organization at All India level to relieve and free the nation from the clutches of British rule and solve the problems of Indians leading to the Birth of the Madras Mahajana Sabha.

Along with the Poona Sarvajanik Sabha, Bombay Presidency Association and the Indian Association, Madras Mahajana Sabha is considered to be a predecessor of the Indian National Congress.

Q Source: Page 141: 12th TN History Textbook

89 The Moderates were called so because 1. They had rejected the idea of participating in legislative councils and instead believed in peaceful protests. 2. They relied on organized struggle against the British, something that was opposite to the extremist camp.

Which of the above is/are correct? 1 onlyA. 2 onlyB. C. Both 1 and 2 NoneD.

Your Answer : Correct Answer : D

Answer Justification :

Justification: The Moderates had faith in the British justice and goodwill. They were called moderates because they adopted peaceful and constitutional means to achieve their demands. They www.insightsias.com 75 TEST - 15(MOCK TEST - 12)

organized in parties and associations, even extremists did the same, but they the latter were more radical in their approach.

Main Demands of Moderates were:

Expansion and reform of legislative councils.

Greater opportunities for Indians in higher posts by holding the ICS examination simultaneously in England and in India.

Separation of the judiciary from the executive.

More powers for the local bodies.

Reduction of land revenue and protection of peasants from unjust landlords.

Abolition of salt tax and sugar duty.

Reduction of spending on army.

Freedom of speech and expression and freedom to form Associations

Q Source: Page 141: 12th TN History Textbook

90 The difference between BS-VI and BS- V Norms is that 1. No particulate matter emissions are allowed in BS-VI unlike BS-V 2. BS-VI norms also add nitrate and sulphur emissions monitoring, something that was voluntary in BS-V norms

Which of the above is/are correct? 1 onlyA. 2 onlyB. C. Both 1 and 2 NoneD.

Your Answer : Correct Answer : D

Answer Justification :

Background: Introduced in the year 2000, the Bharat norms are emission control standards put in www.insightsias.com 76 TEST - 15(MOCK TEST - 12)

place by the government to keep a check on air pollution. Based on the European regulations (Euro norms), these standards set specifications/limits for the release of air pollutants from equipment using internal combustion engines, including vehicles. Typically, the higher the stage, the more stringent the norms.

Justification: The particulate matter emission in BS-V and BS-VI is same for diesel cars though it is 80% less than BS IV.

The nitrogen oxide (NOx) level is, however, 55% less in BS-VI over BS-V which in itself is 28% lower than BS IV.

The sulphur content in fuel norms for diesel and petrol under both BS-V and -VI standards does not change at 10 ppm, though it is substantially less than 50 mandated for both the fuels under BS-IV

Taking into account the serious pollution levels in Delhi and adjoining areas, Petroleum Ministry in consultation with Public Oil Marketing Companies has decided for preponement of BS-VI grade auto fuels in NCT of Delhi w.e.f 01.04.2018 instead of 01.04.2020.

This measure is expected to help mitigate the problem of air pollution in NCT of Delhi and surrounding areas

Q Source: http://indianexpress.com/article/explained/delhi-pollution-smog-bs-vi-emission-norms-for-vehicles-so- near-and-yet-so-far-here-is-why-4945392/

91 Mahatma Gandhi called a halt to the Non-Cooperation Movement due to 1. Motilal Nehru and Chittaranjan Das forming the Swaraj Party amidst the movement 2. that led to violent clashes 3. Gandhi’s arrest by the authorities

Select the correct answer using the codes below. A. 2 and 3 only 2 onlyB. C. 1 and 3 only D. 1, 2 and 3

Your Answer : Correct Answer : B

Answer Justification :

Justification: Statement 2 and 3: The non-cooperation movement was withdrawn because of the Chauri Chaura incident. Although he had stopped the national revolt single-handedly, on March 10, 1922, Gandhi was arrested. On March 18, 1922, he was imprisoned for six years for publishing seditious materials. This led to suppression of the movement and was followed by the arrest of other www.insightsias.com 77 TEST - 15(MOCK TEST - 12)

leaders.

Statement 1: Although most Congress leaders remained firmly behind Gandhi, the determined broke away.

The Ali brothers would soon become fierce critics. Motilal Nehru and Chittaranjan Das formed the Swaraj Party (after the movement, not in between), rejecting Gandhi's leadership.

Many nationalists had felt that the non-cooperation movement should not have been stopped due to isolated incidents of violence, and most nationalists, while retaining confidence in Gandhi, were discouraged.

Q Source: Page 60: 10th NCERT History

92 Consider the following statements. Assertion (A): Recently, the Union Cabinet prohibited export of all types of pulses from India. Reason (R): Indian production of pulses is not sufficient to meet domestic demand.

In the context of the above, which of these is correct? A. A is correct, and R is an appropriate explanation of A. B. A is incorrect, but R is correct. C. A is correct, but R is incorrect. D. Both A and R are incorrect.

Your Answer : Correct Answer : D

Answer Justification :

Justification: Pulses import grew 20 per cent to 5.67 million tonnes in FY17 from FY16. This is because domestic demand is quote high. Due to this, the export of pulses was banned.

But, the Cabinet Committee on Economic Affairs recently gave its approval for removal of prohibition on export of all types of pulses to ensure that farmers have greater choice in marketing their produce and in getting better remuneration for their produce.

Export of pulses would provide an alternative market for the surplus production of pulses. Allowing export of pulses will also help the country and its exporters to regain their markets.

Q Source: AKT/VBA/SH (Release ID :173549)

93 With reference to of 1917, consider the following statements. 1. The word Satyagraha was used for the first time in this agitation. 2. The tinkathia system was associated with this movement 3. and J. B. Kripalani participated in the movement.

Select the correct answer using the codes below. A. 2 and 3 only www.insightsias.com 78 TEST - 15(MOCK TEST - 12)

B. 1 and 2 only 3 onlyC. D. 1, 2 and 3

Your Answer : Correct Answer : A

Answer Justification :

Justification: Statement 1: The word Satyagraha was used for the first time in Anti Rowlatt Act agitation.

Statement 2: Indigo plantations extended into Bihar, where too European planters used the zamindari system to force their peasant tenants to bow to their will.

Where they could not buy zamindaris they obtained leases from local zamindars, and in the form of ‘thekadars’ exercised the same rights over peasants as they would have had as zamindars.

In Champaran district of Bihar, most European planters obtained thekas or leases for whole villages from the large Bettiah zamindari. Here, as the demand for indigo grew with expanding textile imports, the planters imposed what came to be known as the tinkathia system, the peasants being forced to raise indigo on the best parts of their rented lands.

Statement 3: To address the situation, Gandhi arrived in Champaran in 1917 with a team of eminent lawyers: , Rajendra Prasad, Ramnavmi Prasad, and others including J. B. Kripalani.

Q Source: Page 55: 10th NCERT History

94 G. Subramanya Aiyar founded which of the following publications? 1. The Hindu 2. Swadesamitran 3. Madras Express

Select the correct answer using the codes below. A. 1 and 2 only B. 2 and 3 only 3 onlyC. D. 1 and 3 only

Your Answer : Correct Answer : A

Answer Justification :

Justification: He preached nationalism through the Madras Mahajana Sabha. He also founded the The Hindu and Swadesamitran.

www.insightsias.com 79 TEST - 15(MOCK TEST - 12)

He was proprietor, editor and managing director of The Hindu from 20 September 1878 to October 1898.

Subramania Iyer campaigned vehemently for reforms in Hindu society. He supported widow remarriage and desired to abolish untouchability and child marriages.

Q Source: Additional Research: Page 134: 12th TN History Textbook

95 With reference to the East India Association (London), consider the following statements. 1. It was founded by Surendranath Banerjee. 2. It superseded the London Indian Society. 3. It was an extremist organization that had its main goal of organizing an armed struggle against the British in India.

Select the correct answer using the codes below. 1 onlyA. B. 2 and 3 only 2 onlyC. D. 1, 2 and 3

Your Answer : Correct Answer : C

Answer Justification :

Justification: It was founded by Dadabhai Naoroji in 1866, in collaboration with Indians and retired British officials in London.

It superseded the London Indian Society and was a platform for discussing matters and ideas about India, and to provide representation for Indians to the Government.

In 1868, the East India Association had nearly 600 members. This had increased to 1,000 in 1878. Female members were admitted from 1912.

The Association produced a journal (Journal of the East India Association) from its inception which included the papers that were delivered before their meetings.

Papers and proceedings of these meetings were then produced in the Asiatic Quarterly Review, which eventually superseded the Journal of the East India Association.

Q Source: Additional Research: Page 134: 12th TN History Textbook

96 Compensatory Afforestation Fund Management and Planning Authority (CAMPA) was established www.insightsias.com 80 TEST - 15(MOCK TEST - 12)

A. on the directions of the Supreme Court in 2009 B. by a UNEP Project in India in 2012 C. with India’s accession to the Convention on Biological Diversity (CBD) D. after the ratification of the Cartagena and Nagoya protocol

Your Answer : Correct Answer : A

Answer Justification :

Background: Many development and industrial projects such as erection of dams, mining, and construction of industries or roads require diversion of forest land. Any project proponent, government or private must apply for forest clearance from Ministry of Environment and Forests (MoEF), before the conversion of land take place.

This proposal is to be submitted through the concerned forest department of the state government. If clearance is given, then compensation for the lost forest land is also to be decided by the ministry and the regulators.

Due to certain discrepancies in the implementation of compensatory afforestation, some NGOs had approached the SC for relief.

The SC in 2009 issued orders that there will be a Compensatory Afforestation Fund Management and Planning Authority (CAMPA) as National Advisory Council under the chairmanship of the Union Minister of Environment & Forests for monitoring, technical assistance and evaluation of compensatory afforestation activities.

Learning: National CAMPA Advisory Council has been established as per orders of The Hon’ble Supreme Court with the following mandate:

Lay down broad guidelines for State CAMPA.

Facilitate scientific, technological and other assistance that may be required by State CAMPA.

Make recommendations to State CAMPA based on a review of their plans and programmes.

Provide a mechanism to State CAMPA to resolve issues of an inter-state or Centre-State character.

The State CAMPA would presently receive funds collected from user agencies towards compensatory afforestation, additional compensatory afforestation, penal compensatory afforestation, Net Present Value (NPV) and all other amounts recovered from such agencies under the Forest (Conservation) Act, 1980 and presently lying with the Adhoc CAMPA.

Q Source: Protected Areas Conservation: www.insightsias.com 81 TEST - 15(MOCK TEST - 12)

http://www.moef.nic.in/major-initiatives/compensatory-afforestation-fund-management-and-planning -authority-campa

97 The objective(s) of the Deoband Movement was/were to 1. To propagate among the Muslims Western and rational thought 2. To promote women centred social reforms

Which of the above is/are correct? 1 onlyA. 2 onlyB. C. Both 1 and 2 NoneD.

Your Answer : Correct Answer : D

Answer Justification :

Justification: The orthodox section among the Muslim ulema organised the Deoband Moovement.

It was a revivalist movement whose twin objectives were : (i) to propagate among the Muslims the pure teachings of the Koran and the Hadis and (ii) to keep alive the spirit of jihad aganist the foreign rulers.

The new Deoband leader Mahmud-ul-Hasan (1851-1920) sought to impart a political and intellectual content to the religious ideas of the school. The liberal interpretation of Islam created a political awakening among its followers.

Q Source: Page 133: 12th TN History Textbook

98 A species known as dugong (sea cow) is found here. The estuaries of some South Indian rivers drain near the protected area. It is

A. Gulf of Mannar Marine National Park B. Lakshadweep Sanctuary C. Mahatma Gandhi Conservation Park D. Paradip Marine Community Reserve

Your Answer : A Correct Answer : A

Answer Justification :

Learning: The Gulf of Mannar is a large shallow bay forming part of the Laccadive Sea in the Indian Ocean.

The Gulf of Mannar Marine National Park is a protected area of India consisting of 21 small islands www.insightsias.com 82 TEST - 15(MOCK TEST - 12)

(islets) and adjacent coral reefs in the Gulf of Mannar in the Indian Ocean.

The park has a high diversity of plants and animals in its marine, intertidal and near shore habitats. Public access inside the Park is limited to glass bottom boat rides.

Mangroves dominate the intertidal zones of the park islands.

Q Source: Protected areas: Southern India

99 Consider the following chronology and rearrange it suitably. 1. Lahore Congress adopts the demand for Purna Swaraj 2. Second Round Table Conference 3. Ambedkar establishes Depressed Classes Association 4. Civil Disobedience movement relaunched

Select the correct answer using the codes below. 1324A. 1234B. 2413C. 3142D.

Your Answer : Correct Answer : A

Answer Justification :

Justification: These are some of the most important events in Indian freedom struggle post-1918:

www.insightsias.com 83 TEST - 15(MOCK TEST - 12)

We will keep covering them in our tests. Some are already covered in this test.

Q Source: Page 66: 10th NCERT History

100 The term Harijan is often associated with Gandhi because 1. He coined the term. 2. He published a weekly journal of the same name.

Which of the above is/are correct? 1 onlyA. 2 onlyB. C. Both 1 and 2 NoneD.

Your Answer : Correct Answer : B

Answer Justification :

Justification: Statement 1: Though Gandhi popularized the term harijan, which literally meant children of god, it was coined by the Gujarati Bhakti era poet-saint Narsi Mehta, to refer to the children of Devadasis.

According to other source the medieval devotional poet Gangasati used the term to refer to herself during the Bhakti movement, a period in India that gave greater status and voice to women while challenging the legitimacy of caste. Gangasati lived around the 12th-14th centuries and wrote in the Gujarati language.

Statement 2: Gandhi started publishing a weekly journal of the same name on 11 February 1932 from Yerwada Jail during British rule.

Gandhi focussed on social and economic problems in this work, much as his earlier English newspaper, Young India, had done from 1919 to 1932.

Q Source: Additional Research: Page 61: 10th NCERT History

www.insightsias.com 84